NUR 140 Exam #1

अब Quizwiz के साथ अपने होमवर्क और परीक्षाओं को एस करें!

24. The client with ulcerative colitis is scheduled for an ileostomy. The nurse is aware the client's stoma will be located in which area of the abdomen? 1. R Iliac region 2. L Iliac region 3. Epigastric region 4. R Hypochondriac region

1

55. The client diagnosed with acute diverticulitis is complaining of severe abdominal pain. On assessment, the nurse finds a hard rigid abdomen and T 102 degrees Fahrenheit. Which intervention should the nurse implement? 1. Notify the HCP 2. Prepare to administer a Fleet's enema 3. Administer an antipyretic suppository 4. Continue to monitor the client closely

1

57. The client with acute diverticulitis has a nasogastric tube draining green liquid bile. Which intervention should the nurse implement? 1. Document the findings as normal 2. Assess the client's bowel sounds 3. Determine the client's last bowel movement 4. Insert the N/G tube at least 2 more inches

1

20. The client diagnosed with ulcerative colitis has an ileostomy. Which statement indicates the client needs more teaching concerning the ileostomy? 1. "My stoma should be pink and moist." 2. "I will irrigate my ileostomy every morning." 3. "If I get a red, bumpy, itchy rash I will call my HCP." 4. "I will change my pouch if it starts leaking."

2

54. The nurse is preparing to administer the initial does of an aminoglycoside antibiotic to the client just admitted with a diagnosis of acute diverticulitis. Which intervention should the nurse implement? 1. Obtain a serum trough level. 2. Ask about drug allergies. 3. Monitor the peak level. 4. Assess the vital signs.

2

60. The client is admitted to the medical floor with acute diverticulitis. Which collaborative intervention would the nurse anticipate the health-care provider ordering? 1. Administer total parenteral nutrition. 2. Maintain NPO and nasogastric tube. 3. Maintain on a high-fiber diet and increase fluids. 4. Obtain consent for abdominal surgery.

2

14. The client with type 2 diabetes is prescribed prednisone, a steroid, for an acute exacerbation of inflammatory bowel disease. Which intervention should the nurse discuss with the client? 1. Take this medication on an empty stomach 2. Notify the HCP if experiencing a moon face 3. Take the steroid medication as prescribed 4. Notify the HCP if the blood glucose is over 160

3

18. The client is diagnosed with an acute exacerbation of IBD. Which priority intervention should the nurse implement first? 1. Weigh the client daily and document in the client's chart 2. Teach coping strategies such as dietary modifications 3. Record the frequency, amount, and color of stools 4. Monitor the client's oral fluid intake every shift

3

19. The client diagnosed with Crohn's disease is crying and tells the nurse "I can't take it anymore. I never know when I will get sick and end up here in the hospital." Which statement is the nurse's best response? 1. "I understand how frustrating this must be for you." 2. "You must keep thinking about the good things in your life." 3. "I can see you are very upset. I'll sit down and we can talk." 4. "Are you thinking about doing anything like committing suicide?"

3

23. The client diagnosed with ulcerative colitis is prescribed a low-residue diet. Which meal selection indicates the client understands the diet teaching? 1. Grilled hamburger on a wheat bun and fried potatoes. 2. A chicken salad sandwich and lettuce and tomato salad. 3. Roast pork, white rice, and plain custard. 4. Fried fish, whole grain pasta, and fruit salad.

3

51. The client is admitted to the medical unit with a diagnosis of acute diverticulitis. Which health-care provider's order should the nurse question? 1. Insert a nasogastric tube. 2. Start IV D5W at 125 mL/hr. 3. Put client on a clear liquid diet. 4. Place client on bed rest with bathroom privileges.

3

52. The nurse is discussing the therapeutic diet for the client diagnosed with diverticulosis. Which meal indicates the client understands the discharge teaching? 1. Fried fish, mashed potatoes, and iced tea. 2. Ham sandwich, applesauce, and whole milk. 3. Chicken salad on whole-wheat bread and water. 4. Lettuce, tomato, and cucumber salad and coffee.

3

59. The nurse is working in an outpatient clinic. Which client is most likely to have a diagnosis of diverticulosis? 1. A 60-year-old male with a sedentary lifestyle 2. A 72-year-old female with multiple childbirths 3. A 63-year-old female with hemorrhoids 4. A 40-year-old male with a family history of diverticulosis

3

49. The client diagnosed with diverticulitis is complaining of severe pain in the left lower quadrant and has an oral temperature of 100.6 degrees Fahrenheit. Which intervention should the nurse implement first? 1. Notify the HCP 2. Document the findings in the chart 3. Administer an oral antipyretic 4. Assess the client's abdomen

4

50. The nurse is teaching the client diagnosed with diverticulosis. Which instruction should the nurse include in the teaching session? 1. Discuss the importance of drinking 1,000mL of water daily 2. Instruct the client to exercise at least three (3) times a week 3. Teach the client about eating a low-residue diet 4. Explain the need to have daily bowel movements

4

53. The client is two (2) hours post-colonoscopy. Which assessment data would warrant intermediate intervention by the nurse? 1. The client has a soft, nontender abdomen. 2. The client has a loose, watery stool. 3. The client has hyperactive bowel sounds. 4. The client's pulse is 104 and BP is 98/60.

4

How long is the drug regimine for TB?

9 months - 1 year

Normal Chlorine level

96-106

A patient is being transferred to your unit with acute cholecystitis. In report the transferring nurse tells you that the patient has a positive Murphy's Sign. You know that this means:* A. The patient stops breathing in when the examiner palpates under the ribs on the right upper side of the abdomen at the midclavicular line. B. The patient stops breathing out when the examiner palpates under the ribs on the right upper side of the abdomen at the midclavicular line. C. The patient verbalizes pain when the lower right quadrant is palpated. D. The patient reports pain when pressure is applied to the right lower quadrant but then reports an increase in pain intensity when the pressure is released.

A

An older adult resident in a long-term-care facility becomes confused and agitated, telling the nurse, "Get out of here! You're going to kill me!" Which action will the nurse take first? A. Check the resident's oxygen saturation. B. Do a complete neurologic assessment. C. Give the prescribed PRN lorazepam (Ativan). D. Notify the resident's primary care provider

A A common reason for sudden confusion in older clients is hypoxemia caused by undiagnosed pneumonia. The nurse's first action should be to assess oxygenation by checking the pulse oximetry. Determining the cause of the confusion is the primary goal of the RN.

A pt is being admitted for pneumonia. The sputum culture is positive for streptococcus, and the pt asks the nurse how long the treatment will be. The nurse knows? A. The pt will be treated for 5 to 7 days. B. The pt will require IV antibiotics for 7 to 10 days. C. The pt will complete 6 days of therapy. D. The pt must be afebrile for 24 hours.

A Anti-infectives usually are used for 5 to 7 days in uncomplicated community-acquired pneumonia, and for up to 21 days in an immunocompromised pt or one with hospital-acquired pneumonia

A critical concern for a post op pt returning to the floor is r/t impaired oxygenation caused by inadequate ventilation. The ABG and assessment finding that alerts the nurse to use oxygen and the ICS is A. PaO2 is 90mm Hg with crackles B. PaO2 is 45mm Hg with atelectasis C. PaO2 is 90mm Hg with wheezing D. PaO2 is 38mm Hg with clear lung sounds

A PaO2 is 90mm Hg with crackles

A nurse is caring for an 89-year-old client admitted with pneumonia. He has an IV of normal saline running at 100 mL/hr and antibiotics that were initiated in the emergency department 3 hours ago. He has oxygen at 2 liters/nasal cannula. What assessment finding by the nurse indicates that goals for a priority diagnosis have been met for this client? A. The client is alert and oriented to person, place, and time. B. Blood pressure is within normal limits and client's baseline. C. Skin behind the ears demonstrates no redness or irritation. D. Urine output has been >30 mL/hr per Foley catheter.

A Rationale: One of the first manifestations of pneumonia in an older adult is acute confusion as a result of impaired gas exchange. A client with pneumonia who is alert and oriented to person, place, and time is responding well to appropriate therapy for the disorder. The blood pressure is not an indicator of effective management of pneumonia, and neither is urine output. The skin behind his ears being intact is important and desirable but is not an outcome indicator for pneumonia management.

A pt with type 2 diabetes and cirrhosis asks if it would be okay to take silymarin (milk thistle) to help minimize liver damage. The nurse responds based on what knowledge? a. milk thistle may affect liver enzymes and thus alter drug metabolism b. milk thistle is generally safe in recommended doses for up to 10 yrs c. there is unclear scientific evidence for the use of milk thistle in treating cirrhosis d. milk thislte may elevate the serum glucose level and is thus contraindicated in diabetes.

A There is evidence that there is no real benefit from using milk thistle to protect liver enzyme from toxic damage. Milk thistle does affect liver enzyme and could alter drug metabolism. Pt will need to be monitored for drug interactions. It is dafe for up to 6 yrs not 10, and it may lower, not elevate, blood glucose levels.

The pt with cirrhosis is being taught self care. which statement indicates the pt needs MORE teaching? a. if i notice a fast heart rate and irregular beats, this is normal for cirrhosis b. i need to take good care of my belly and ankle skin where it is swollen. c. a scrotal support may be more comfortable when i have scrotal edema d. i can use pillows to support my head to help me breathe when i am in bed.

A This may indicative of hypokalemia and should be reported to healthcare provider, this is not normal for cirrhosis.

When auscultating the chest of a client with pneumonia, the nurse would expect to hear which of the following sounds over areas of consolidation? A. Bronchial B. Bronchovesicular C. Tubular D. Vesicular

A Bronchial Chest auscultation reveals bronchial breath sounds over areas of consolidation. Bronchovesicular are normal over midlobe lung regions, tubular sounds are commonly heard over large airways, and vesicular breath sounds are commonly heard in the bases of the lung fields.

Which pneumonia complication does the nurse recognize as creating pain that increases on inspiration because of inflammation of the parietal pleura? A. pleuritic CP B. meningitis C. COPD D. Pulmonary emboli

A Pleuritic CP

A nurse is auscultating the lower lung fields of a pneumonia pt. She hears coarse crackles and ids the problem as impaired oxygenation. She knows the underlying physiologic condition associated with pt condition is A. hypoxemia B. hyperemia C. hypocapnia D. hypercapnia

A hypoxemia

A nurse is assessing a client who has Crohn's disease. Which of the following findings should the nurse expect? A. Fatty diarrheal stools B. Hyperkalemia C. Weight gain D. Sharp epigastric pain

A Fatty diarrheal stools Steatorrhea, or fatty stool, is an expected finding in a client who has Crohn's disease.

When caring for a pt with liver disease, the nurse recognizes the need to prevent bleeding resulting from altered clotting factors and rupture of varices. Which nursing interventions would be appropriate to achieve this outcome? Select all that apply. a. use smallest gauge needle possible when giving injections or drawing blood. b. teach pt to avoid straining at stool, vigorous blowing of nose, and coughing c. advise pt to use soft-bristle toothbrush and avoid ingestion of irritating food. d. apply gentle pressure for the shortest possible time period after performing venipuncture e. instruct pt to avoid aspirin and NSAIDs to prevent hemorrhage when varices are present.

A, B, C, E - small gauge minimize risk of bleeding into tissues. - avoiding strain reduces hemorrhage - soft bristle reduce injury to highly vascular mucous membranes - apply gentle but prolonged pressure to venipuncture - aspirin and NSAIDs should not be used in pt with liver disease b/c they interfere w/ platelet aggregation, increasing bleeding risk

The nurse knows, in caring for her ventilated pt, that the goal is to prevent VAP. She identifies the following as important in reaching this goal. Select All That Apply A. she should avoid wearing jewelry B. HOB should be up C. Administer chest physiotherapy D. Provide oral care every 12 hours E. Hand hygeine

A, B, E chest physiotherapy is not an intervention for this pt oral care should be provided every 8 hours, not 12 hours

Which conditions does the nurse recognize as a risk for developing aspiration pneumonia? Select All That Apply A. continuous tube feed B. bronchoscopy procedure C. MRI D. decreased LOC E. stroke F. chest tube

A, B. D. E continuous tube feed bronchoscopy procedure decreased LOC stroke

A nurse is assessing a client who has appendicitis. Which of the following findings should the nurse expect? (Select all that apply) A. Oral temperature 38.4° C (101.1° F) B. WBC 6,000/mm3 C. Bloody diarrhea D. Nausea and vomiting E. Right lower quadrant pain

A, D, E Oral temperature 38.4° C (101.1° F) is correct. A low-grade temperature is an expected finding in a client who has appendicitis. Nausea and vomiting is correct. Nausea and vomiting are expected findings in a client who has appendicitis. Right lower quadrant pain is correct. Right lower quadrant pain is an expected finding in a client who has appendicitis.

A nurse is caring for a client who has ulcerative colitis. The client has had several exacerbations over the past 3 yr. Which of the following instructions should the nurse include in the plan of care tom inimize risk of further exacerbations? (Select all that apply) A. Use progressive relaxation techniques. B. Increase dietary fiber intake. C. Drink two 240 mL (8 oz) glasses of milk per day. D. Arrange activities to allow for daily rest periods. E. Restrict intake of carbonated beverages.

A, D, E Use progressive relaxation techniques is correct. Progressive relaxation techniques, a form of biofeedback, are recommended to help the client minimize stress, which can precipitate an exacerbation. Arrange activities to allow for daily rest periods is correct. Daily rest periods decrease stress and reduce intestinal motility. Restrict intake of carbonated beverages is correct. The client should avoid gastrointestinal stimulants, such as carbonated beverages, nuts, peppers, and smoking.

A pt is seen in the HCP office and dx with community-acquired pneumonia. The nurse knows the most common symptoms that this pt may have is A. dyspnea B. abdominal pain C. back pain D. hypoxemia E. chest discomfort F. a smoker

A, D, E dyspnea hypoxemia chest discomfort

A nurse is completing nutrition teaching for a client who has pancreatitis. Which of the following statements by the client indicates an understanding of the teaching? (Select all that apply.) A. "I plan to eat small, frequent meals." B. "I will eat easy-to-digest foods with limited space." C. "I will use skim milk when cooking." D. "I plan to drink regular cola." E. "I will limit alcohol intake to two drinks per day."

A. "I plan to eat small, frequent meals." B. "I will eat easy-to-digest foods with limited space." C. "I will use skim milk when cooking."

A nurse is providing discharge teaching for a client who has peptic ulcer disease and a new famotidine. Which of the following statements by the client indicates an understanding of the teaching? A. "I should take this medication at bedtime." B. "I should expect this medication to discolor my stools." C. "I will drink iced tea with my meals and snacks." D. "I will monitor my blood glucose level regularly while taking this medication."

A. "I should take this medication at bedtime." The nurse should instruct the client to take the medication at bedtime to inhibit the action of histamine at the H2-receptor site in the stomach.

A nurse is providing discharge teaching for a client who has GERD. Which of the following statements by the client indicates an understanding of the teaching? A. "I will decrease the amount of carbonated beverages I drink." B. "I will avoid drinking liquids for 30 minutes after taking a chewable antacid tablet." C. "I will eat a snack before going to bed." D. "I will lie down for at least 30 minutes after eating each meal."

A. "I will decrease the amount of carbonated beverages I drink." The nurse should instruct the client to limit or eliminate fatty foods, coffee, cola, tea, carbonated beverages, and chocolate from his diet because they irritate the lining of the stomach.

A nurse in the emergency department is assessing a client for a closed pneumothorax and significant bruising of the left chest following a motor-vehicle crash. The client reports severe left chest pain on inspiration. The nurse should assess the client for which of the following manifestations of pneumothorax? A. Absence of breath sounds B. Expiratory wheezing C. Inspiratory stridor D. Ronchi

A. Absence of breath sounds - A client who has pneumothorax experiences severely diminished or absent breath sounds on the affected side.

The nurse is caring for a patient who has developed a bradycardia. Which possible causes does the nurse investigate? (Select all that apply.) A. Bearing down for a bowel movement B. Possible inferior wall myocardial infarction (MI) C. Patient stating that he just had a cup of coffee D. Patient becoming emotional when visitors arrived E. Diltiazem (Cardizem) administered 1 hour ago

A. Bearing down for a bowel movement B. Possible inferior wall myocardial infarction (MI) E. Diltiazem (Cardizem) administered 1 hour ago

A nurse is reviewing nutrition teaching for a client who has cholecystitis. The nurse should identify that which of the following food choices can trigger cholecystitis? A. Brownie with nuts B. Bowl of mixed fruit C. Grilled turkey D. Baked potato

A. Brownie with nuts

A client has undergone the Whipple procedure (radical pancreaticoduodenectomy) for pancreatic cancer. Which precautionary measures does the nurse implement to prevent potential complications? Select all that apply. A. Check blood glucose often. B. Check bowel sounds and stools. C. Ensure that drainage color is clear. D. Monitor mental status. E. Place the client in the supine position.

A. Check blood glucose often. B. Check bowel sounds and stools. D. Monitor mental status.

A client who was awaiting liver transplantation is excluded from the procedure after the presence of which condition is discovered? A. Colon cancer with metastasis to the liver B. Hypertension C. Hepatic encephalopathy D. Ascites and shortness of breath

A. Colon cancer with metastasis to the liver

The nurse is caring for a patient with advanced heart failure who develops asystole. The nurse corrects the graduate nurse when the graduate offers to perform which intervention? A. Defibrillation B. Cardiopulmonary resuscitation (CPR) C. Administration of epinephrine D. Administration of oxygen

A. Defibrillation

A nurse is caring for a client who has cirrhosis. Which of the following medications can the nurse expect to administer to this client? (Select all that apply.) A. Diuretic B. Beta-blocking agent C. Opioid analgesic D. Lactulose E. Sedative

A. Diuretic B. Beta-blocking agent D. Lactulose

A nurse in an urgent care clinical is collecting data from a patient who reports exposure to anthrax. Which of the following findings is an indication of the prodromal stage of inhalation anthrax? A. Dry cough B. Rhinitis C. Sore throat D. Swollen lymph nodes

A. Dry cough A dry cough is a clinical manifestation found in the prodromal stage of having inhalation anthrax. During this stage, it is difficult to distinguish from influenza or pneumonia because there is no sore throat or rhinitis.

A nurse is planning care for a client who has chronic obstructive pulmonary disease and is malnourished. Which of the following recommendations to promote nutritional intake should the nurse include in the plan? A. Eat high-calorie foods first B. Increase intake of water at meal times C. Perform active range-of-motion exercises before meals D. Keep saltine crackers nearby for snacking

A. Eat high-calorie foods first - Although it is important for a client who has COPD to maintain adequate fluid intake to prevent dehydration and inhibit the production of tenacious secretions, the client should limit intake of water at mealtimes to reduce the feeling of early satiety.

What teaching does the nurse include for a patient with atrial fibrillation who has a new prescription for warfarin? A. "It is important to consume a diet high in green leafy vegetables." B. "You would take aspirin or ibuprofen for headache." C. "Report nosebleeds to your provider immediately." D. "Avoid caffeinated beverages."

C. "Report nosebleeds to your provider immediately."

A client diagnosed with acalculous cholecystitis asks the nurse how the gallbladder inflammation developed when there is no history of gallstones. What is the nurse's best response? A. "This may be an indication that you are developing sepsis." B. "The gallstones are present, but have become fibrotic and contracted." C. "This type of gallbladder inflammation is associated with hypovolemia." D. "This may be an indication of pancreatic disease."

C. "This type of gallbladder inflammation is associated with hypovolemia."

A nurse in a clinic is providing teaching for a client who is to have a tuberculin skin test. Which of the following information should the nurse include? A."If the test is positive, it means you have an active case of tuberculosis." B. "If the test is positive, you should have another tuberculin skin test in 3 weeks." C. "You must return to the clinic to have the test read in 2 or 3 days." D. "A nurse will use a small lancet to scratch the skin of your forearm before applying the tuberculin substance."

C. "You must return to the clinic to have the test read in 2 or 3 days." - The client should have the skin test read in 2 to 3 days. An area of induration after 48 to 72 hr indicates exposure to the tubercle bacillus. If the client does not return to have the test read within 72 hr, another tuberculin skin test is necessary.

When caring for a client with portal hypertension, the nurse assesses for which potential complications? Select all that apply. A. Esophageal varices B. Hematuria C. Fever D. Ascites E. Hemorrhoids

A. Esophageal varices D. Ascites E. Hemorrhoids

The nurse is caring for a patient with atrial fibrillation (AF). In addition to an antidysrhythmic, what medication does the nurse plan to administer? A. Heparin B. Atropine C. Dobutamine D. Magnesium sulfate

A. Heparin

Which waveform indicates proper function of the sinoatrial (SA) node? A. The QRS complex is present. B. The PR interval is 0.24 second. C. A P wave precedes every QRS complex. D. The ST segment is elevated.

C. A P wave precedes every QRS complex.

Which diagnostic results lead the nurse to suspect that a client may have gallbladder disease? A. Increased white blood cell (WBC) count, visualization of calcified gallstones, edema of the gallbladder wall B. Decreased WBC count, visualization of calcified gallstones, increased alkaline phosphatase C. Increased WBC count, visualization of noncalcified gallstones, edema of the gallbladder wall D. Decreased WBC count, visualization of noncalcified gallstones, increased alkaline phosphatase

A. Increased white blood cell (WBC) count, visualization of calcified gallstones, edema of the gallbladder wall

A client is experiencing an attack of acute pancreatitis. Which nursing intervention is the highest priority for this client? A. Measure intake and output every shift. B. Do not administer food or fluids by mouth. C. Administer opioid analgesic medication. D. Assist the client to assume a position of comfort.

C. Administer opioid analgesic medication.

A nurse is assessing a client who has acute hepatitis B. Which of the following findings should the nurse expect? A. Joint pain B. Obstipation C. Abdominal distention D. Periumbilical discoloration

A. Joint pain Joint pain is an expected finding in a client who has acute hepatitis B.

When caring for a client with hepatic encephalopathy, in which situation does the nurse question the use of neomycin (Mycifradin)? A. Kidney failure B. Refractory ascites C. Fetor hepaticus D. Paracentesis scheduled for today

A. Kidney failure

A nurse is teaching a client who has hepatitis B about home care. Which of the following instructions should the nurse include in the teaching? (Select all that apply.) A. Limit physical activity B. Avoid alcohol C. Take acetaminophen for comfort D. Wear a mask when in public places E. Eat small frequent meals

A. Limit physical activity B. Avoid alcohol E. Eat small frequent meals

A client who had been hospitalized with pancreatitis is being discharged with home health services. The client is severely weakened after this illness. Which nursing intervention is the highest priority in conserving the client's strength? A. Limiting the client's activities to one floor of the home B. Instructing the client to take an as-needed (PRN) sleeping medication at night C. Arranging for the client to have a nutritional consult to assess the client's diet D. Asking the health care provider for a request for PRN nasal oxygen

A. Limiting the client's activities to one floor of the home

A nurse is developing a plan of care for a client who has cirrhosis and ascites. Which of the following inteventions should the nurse include the plan? A. Measure the client's abdominal girth daily B. Check mental status once daily C. Provide a daily intake of 4g of sodium for the client D. Assess the client's breath sounds every 12 hr

A. Measure the client's abdominal girth daily The nurse should measure the client's abdominal girth and weigh the client daily to monitor the amount of fluid accumulation in the abdomen and the effectiveness of treatment measures.

A nurse is planning care for a client who has a new prescription for TPN. Which of the following interventions should be included in the plan of care? (Select all that apply.) A. Obtain a capillary blood glucose four times daily. B. Administer prescribed medications through a secondary port on the TPN IV tubing. C. Monitor vital signs three times during the 12-hour shift. D. Change the TPN IV tubing every 24 hr. E. Ensure a daily aPTT is obtained.

A. Obtain a capillary blood glucose four times daily. C. Monitor vital signs three times during the 12-hour shift. D. Change the TPN IV tubing every 24 hr.

The nurse is caring for a patient with heart rate of 143 beats/min. For which manifestations does the nurse observe? (Select all that apply.) A. Palpitations B. Increased energy C. Chest discomfort D. Flushing of the skin E. Hypotension

A. Palpitations C. Chest discomfort E. Hypotension

When caring for a client with Laennec's cirrhosis, which of these does the nurse expect to find on assessment? Select all that apply. A. Prolonged partial thromboplastin time B. Icterus of skin C. Swollen abdomen D. Elevated magnesium E. Currant jelly stool F. Elevated amylase level

A. Prolonged partial thromboplastin time B. Icterus of skin C. Swollen abdomen

A nurse is caring for a client who is receiving TPN solution. The current bag of solution was hung 24 hr ago, and 400 mL remains to infuse. Which of the following is the appropriate action for the nurse to take? A. Remove the current bag and hang a new bag. B. Infuse the remaining solution at the current rate and then hang a new bag. C. Increase the infusion rate so the remaining solution is administered within the hour and hang a new bag. D. Remove the current bag and hang a bag of lactated Ringer's.

A. Remove the current bag and hang a new bag.

A nurse on a medical-surgical unit is caring for a client who is postoperative following a hip replacement surgery. The client reports feeling apprehensive and restless. Which of the follow findings should the nurse recognize as an indication of a pulmonary embolism (PE)? A. Sudden onset of dyspnea B. Tracheal deviation C. Bradycardia D. Difficulty swallowing

A. Sudden onset of dyspnea Clinical manifestations of a PE have a rapid onset. Dyspnea occurs due to reduced blood flow to the lungs.

Which patient is appropriate for the cardiac care unit charge nurse to assign to the float RN from the medical-surgical unit? A. The 64-year-old patient admitted for weakness who has a sinus bradycardia with a heart rate of 58 beats/min B. The 71-year-old patient admitted for heart failure who is short of breath and has a heart rate of 120 to 130 beats/min C. The 88-year-old patient admitted with an elevated troponin level who is hypotensive with a heart rate of 96 beats/min D. The 92-year-old patient admitted with chest pain who has premature ventricular complexes and a heart rate of 102 beats/min

A. The 64-year-old patient admitted for weakness who has a sinus bradycardia with a heart rate of 58 beats/min

The nurse is caring for a client who has cirrhosis of the liver. The client has exhibited hand flapping and mental confusion for several weeks. Although the mental confusion is worsening, the client has stopped exhibiting hand flapping movements. How will the nurse interpret these findings? A. The client's symptoms are progressing and getting worse. B. The client's serum ammonia levels are decreasing. C. The client probably has a decrease in serum proteins. D. The client is showing signs of improvement.

A. The client's symptoms are progressing and getting worse.

A nurse is providing teaching to a patient about pulmonary function tests. Which of the following tests measures the volume of air the lungs can hold at the end maximum inhalation? A. Total lung capacity B. Vital lung capacity C. Functional residual capacity D. Residual volume

A. Total lung capacity Pulmonary function tests are used to examine the effectiveness of the lungs and identify lung problems. Total lung capacity measures the amount of air the lungs can hold after maximum inhalation.

After receiving change-of-shift report on these clients, which client does the nurse plan to assess first? A. Young adult client with acute pancreatitis who is dyspneic and has a respiratory rate of 34 to 38 breaths/min B. Adult client admitted with cholecystitis who is experiencing severe right upper quadrant abdominal pain C. Middle-aged client who has an elevated temperature after undergoing endoscopic retrograde cholangiopancreatography D. Older adult client who is receiving total parenteral nutrition after a Whipple procedure and has a glucose level of 235 mg/dL (13.1 mmol/L)

A. Young adult client with acute pancreatitis who is dyspneic and has a respiratory rate of 34 to 38 breaths/min

Elevated serum sodium (hypernatremia)

Acute hypernatremia often results in significant brain shrinkage, thus causing mechanical traction of cerebral vasculature. Stretching of bridging veins can result in subdural hemorrhages. Venous congestion can lead to thrombosis of the intracranial venous sinuses.

The residents and staff of a nursing home are in a geographical area with reports of confirmed influenza cases. The nurse administrator has decided to offer antiviral medication to all staff and residents as a preventive measure. When is it safe for them to stop taking the​ drugs?

After 7 days with no new flu cases- correct After 5 consecutive days of medication administration After the recipients get immunity to the flu After no reported deaths due to flu in 2 days Rationale After 7 days with no new flu​ cases, the prophylaxis can stop. That is more than 5 consecutive days of medications. An absence of reported deaths is not a signal to stop. The antivirals do not confer immunity to the flu.

A nurse is caring for a patient who presents to the emergency department with abdominal trauma and requires advanced trauma life support (ATLS). Arrange the order in which the nurse should perform the task. Exposure Breathing Disability Airway Circulation

Airway Breathing Circulation Disability Exposure

Do patients with cirrhosis tend to be more alkalotic or acidotic?

Alkalotic because those with cirrhosis tend to have hyperventilation, making them alkalotic

The nurse is assessing a client with tuberculosis. Which should the nurse focus on during this​ assessment? (Select all that​ apply.) A. Presence of cough B. Difficulty breathing C. Skin color D. Carbon dioxide level E. Nasal congestion

Answer: A, B, C

The nurse is reviewing data collected during the assessment of a client with tuberculosis. Which nursing diagnosis should the nurse select for this​ client? (Select all that​ apply.) A. Infection, Risk for B. Health: Community, Deficient C. Resilience, Impaired D. Fatigue E. Resilience, Impaired

Answer: A, B, C, D

The nurse instructs a client with tuberculosis on prescribed medication. Which finding should the nurse instruct the client to report to the healthcare​ provider? (Select all that​ apply.) A. Yellow tint to sclera B. Sudden weight gain C. Hemoptysis D. Orange tint to sweat E. Chest pain

Answer: A, B, C, E

The nurse in a community clinic is asked to determine which clients require tuberculosis testing. Which individual should the nurse recommend for this​ screening? (Select all that​ apply.) A. An individual with close contacts who already have or are suspected to have tuberculosis B. An individual who is a resident or staff member of a​ long-term residential facility C. An individual who had the bacille ​Calmette-Guérin ​(BCG) vaccine D. An individual that has had PPD E. An individual infected with HIV or at high risk for HIV infection

Answer: A, B, E

1. The function of the liver includes which of the following? (Select all that apply.) A. Metabolism of proteins B. Production and secretion of bile salts C. Production of vitamins A B D E K D. Filters blood of toxins E. Production of red blood cells

Answer: A, B, and D

The nurse is caring for a child diagnosed with tuberculosis infection. Which risk factor should the nurse identify that would greatly increase the risk for progression to​ disease? (Select all that​ apply.) A. Presence of HIV infection B. Genetic factors C. Age less than 2 years D. Virulence of the organism E. Magnitude of the infection

Answer: A, C

The nurse instructs a client with tuberculosis on the medication rifampin. Which client statement indicates teaching has been​ effective? (Select all that​ apply.) A. "I should take rifampin on an empty​ stomach." B. "I need to monitor my vision daily by reading a​ newspaper." C. "I should not take aspirin while I am taking​ rifampin." D. "I should not be frightened if my urine changes to an​ orange-red color; it is a normal side​ effect." E. "I need to take pyridoxine​ (vitamin B6) along with the​ rifampin."

Answer: A, C, D

The nurse is preparing teaching for a client newly diagnosed with tuberculosis. Which drug generally used in initial treatment should the nurse include in the​ session? (Select all that​ apply.) A. Isoniazid B. Amikacin C. Pyrazinamide D. Rifampin E. Ethambutol

Answer: A, C, D, E

The nurse is providing care to a client who has been diagnosed with tuberculosis. Which diagnostic test should the nurse expect to be prescribed prior to initiating antibiotic​ treatment? (Select all that​ apply.) A. Polymerase chain reaction​ (PCR) B. Intradermal PPD​ (Mantoux) test C. Sputum culture D. Tine test E. Sputum smear

Answer: A, C, E

The nurse is preparing a plan of care for a client diagnosed with tuberculosis. Which goal and outcome should the nurse identify for this​ client? (Select all that​ apply.) A. The client will demonstrate behaviors that reduce the risk of spreading the disease to others. B. The client with active tuberculosis complies with prescribed​ therapies, symptoms​ resolve, and chest​ x-rays improve. C. The client with latent infection completes therapy and does not develop active tuberculosis. D. The client will have the resources necessary to obtain required supplies and medications. E. The client will articulate required treatment and​ follow-up care.

Answer: A, D, E

3. The liver receives what percentage of cardiac output? A. 10% B. 25% C. 40% D. 50%

Answer: B

An older adult client experiencing a​ cough, hemoptysis, night​ sweats, anorexia, and weakness reports being told of having tuberculosis when younger. Which reason should the nurse suspect is responsible for the​ client's current​ symptoms? A. New-onset tuberculosis B. Reactivation tuberculosis C. Skeletal tuberculosis D. Dormant tuberculosis

Answer: B

4. A patient with liver disease is being evaluated for varices. The nurse prepares the patient for which procedure? A. Liver biopsy B. EGD C. ERCP D. TIPS

Answer: B Rationale: An EGD is a diagnostic procedure where an endoscope is placed through the patient's mouth into the GI tract to visualize upper GI bleeding caused by esophageal varices. While the other procedures listed are oftentimes used in patients with liver failure, they are not used to evaluate varices.

1. The nurse correlates which laboratory data to decreased liver function secondary to liver cancer or cirrhosis? A. Elevated serum globulin level B. Elevated serum ammonia level C. Elevated serum protein level D. Elevated serum amylase

Answer: B Rationale: Laboratory data collected to determine liver dysfunction includes an elevate serum ammonia level. Elevated amylase indicates pancreatitis, and elevated serum globulins and proteins may be indicative of nutritional issues or other diseases non liver related.

3. A patient just returned from the procedure area after having a liver biopsy. The primary nursing intervention includes which action? A. Ambulation B. Monitor blood pressure and assess the site for bleeding C. Heparin drip D. Obtain a culture from the biopsy site

Answer: B Rationale: Nursing interventions post-liver biopsy include blood pressure monitoring and site assessment as there is the potential for bleeding post procedure due to the vascularity of the liver.

In a patient with cirrhosis, the nursing diagnosis "risk for injury and bleeding related to prolonged clotting factors" is most appropriate related to which disorder? A. Pruritus B. Vitamin K deficiency C. Hyponatremia D. Ascites

Answer: B Rationale: Patients with cirrhosis have a decrease in the synthesis of bile salts. The liver makes clotting factors which rely on the presence of vitamin K to function normally. Vitamin K is a fat soluble vitamin that requires bile salts to be absorbed properly. In cirrhosis there is a decrease in the bile salts and decreased ability to absorb vitamin K.

Which statement by a patient diagnosed with liver trauma indicates understanding of the prescribed plan of care? A. "I will need a liver transplant." B. "I will need a blood transfusion." C. "I am at increased risk for infection." D. "I will never be able to drink alcohol again."

Answer: B Rationale: The liver is a highly vascular organ. Traumatic injuries to the liver include lacerations, contusions, and hematomas. Patients require fluid resuscitation, frequent hemoglobin monitoring, and the possibility of blood transfusion.

The nurse is planning care for a client who is homeless. The client is prescribed four drugs to treat tuberculosis. Which action should the nurse take to ensure compliance with this medication​ therapy? (Select all that​ apply.) A. Ask the healthcare provider to consider hospitalizing the client for initial treatment to ensure compliance. B. Work collaboratively with other healthcare team members to identify barriers or challenges. C. Tailor teaching concerning the drugs to the needs of the client. D. Assess the​ client's understanding of the disease​ process, and identify misperceptions and emotional reactions. E. Reduce the number of drugs and the duration of taking the medicine to accommodate the​ client's transient situation.

Answer: B, C, D

The nurse suspects that a client is at risk for tuberculosis. Which risk factor should the nurse assess in this​ client? (Select all that​ apply.) A. Sharing clothes with an infected individual B. Living in a poorly ventilated environment C. Using injection drugs D. Being an immigrant to the United States E. Having a compromised immune system

Answer: B, C, D, E

The infection control nurse is teaching the staff at a​ long-term care facility after a recent outbreak of tuberculosis. Which element of infection control should the nurse include in the​ teaching? (Select all that​ apply.) A. Implementation of universal screening B. Use of airborne precautions C. Treatment of clients with suspected or confirmed disease D. Administration of the bacille ​Calmette-Guérin ​(BCG) vaccine to residents E. Identification of infected individuals

Answer: B, C, E

The nurse is identifying interventions for a client with tuberculosis. Which nursing intervention should the nurse identify to address the risk of infecting​ others? (Select all that​ apply.) A. Providing verbal and written instructions about when to take the medications B. Informing all personnel who have contact with the client of the diagnosis C. Teaching the client how to avoid transmitting the disease to others D. Assessing​ self-care abilities and support systems E. Teaching the client why it is important to comply with prescribed treatments for the whole course of therapy

Answer: B, C, E

The nurse assesses for which clinical manifestations in the patient diagnosed with liver cancer? (Select all that apply.) A. Periumbilical pain B. Anorexia C. Hemoptysis D. Fatigue E. Jaundice

Answer: B, D, and E Rationale: Because the tumor typically impairs normal functions of the liver, the patients exhibit abdominal pain, weight loss, anorexia, weakness, fatigue, jaundice, and ascites. Pain is typically in the right upper quadrant. Hemoptysis is associated with bleeding esophageal varices

The public health nurse is training a nurse on tuberculin skin testing. Which information about the Mantoux test should the public health nurse include in the​ training? (Select all that​ apply.) A. "PPD (0.1​ mL) is injected intradermally into the dorsal aspect of the​ forearm." B. "The test is read within 48 to 72​ hours." C. "This test is less accurate than the​ T-SPOT test." D. "Ten tuberculin units are​ injected." E. "Diameter of induration is recorded in​ millimeters."

Answer: B, E

The nurse visits the home of a client with tuberculosis. Which action should the nurse teach family members to take during the first 2 weeks of treatment to prevent the spread of the infection to other family​ members? A. Be compliant with the medication regimen. B. Ensure that housemates of the client are tested and receive prophylactic treatment if indicated. C. Use disposable tissues to contain respiratory secretions. D. Emphasize the importance of maintaining good general health through diet and exercise.

Answer: C

Elevated ammonia levels can lead to hepatic encephalopathy. Which provider order best reduces this risk in patients with cirrhosis? A. Administer furosemide and spironolactone. B. Administer antibiotics. C. Restrict protein intake. D. Restrict caloric intake.

Answer: C Rationale: Encephalopathy is caused by accumulation of toxins in the liver, one of which is ammonia. Ammonia is produced when proteins are broken down by the body. In a patient with cirrhosis the liver is not able to clear this toxin from the body thus, it is important to restrict protein intake.

5. A 17-year-old patient sustained blunt abdominal trauma (was hit in the abdomen with a baseball bat) and is being discharged home after 24 hours of observation. Discharge planning includes which teaching? A. Self-defense maneuvers B. Antibiotic teaching C. Education about abdominal signs and symptoms of increased pain, tenderness D. Limit solid food intake

Answer: C Rationale: It is important that the patient understand that increased pain or tenderness after liver trauma may indicate a complication from the trauma and should be reported immediately to the health-care provider

In reviewing diagnostic results of a patient with suspected hepatitis, the nurse correlates which result as consistent with hepatitis A? A. Prolonged prothrombin time (PT) B. Decreased white blood cell count C. Presence of IgM anti-HAV D. Detectable serum HBV DNA

Answer: C Rationale: The most definitive diagnosis of Hepatitis A is through a blood test for the presence of antihepatitis A immunoglobulin M (IgM anti-HAV). Acute hepatitis B occurs after an incubation period of 4 to 10 weeks, and chronic hepatitis B is diagnosed by detectable serum HBV-DNA levels and persistent elevation of ALT and AST levels.

The nurse recognizes which patient is at greatest risk for type 1 autoimmune hepatitis? A. A 45-year-old postmenopausal female B. A 30-year-old female with a history of hyperthyroidism C. A 16-year-old female with type 1 diabetes mellitus D. A 12-year-old female with autism

Answer: C Rationale: Type 1 autoimmune hepatitis occurs most frequently in females during adolescence, and these patients usually have other autoimmune disorders such as type 1 diabetes, Graves' disease, ulcerative colitis, or proliferative glomerulonephritis. Type 2 autoimmune hepatitis is less common and affects girls between ages 2 and 14.

2. The nurse caring for a patient with liver failure notes a change in mental status and elevated ammonia level. Which ordered interventions are most directly related to treating these clinical manifestations? A. Diuretics B. High-protein diet C. Coumadin D. Lactulose and neomycin

Answer: D Rationale: Interventions to decrease the build up of ammonia in the body include laxatives and antibiotic therapy. The antibiotic neomycin helps decrease the amount of bacteria, and the laxative helps remove the toxins from the body. Protein restricted diets are important with liver failure diuretics are used in liver failure for fluid overload, not increased ammonia levels and Coumadin is a blood thinning agent, not utilized in a patient for elevated ammonia levels.

Treatment for influenza

Antipyretics (acetemenophen) Analgesics Antivirals: Rapivab (peramivir) Relenza (zanamivir) Tamiflu (oseltamivir phosphate, also available as generic) Xofluza (baloxavir marboxil)

The nurse is reviewing the record of a client with a dx of cirrhosis and notes that there is documentation of the presence of asterixis. How should the nurse assess for its presence? Dorsiflex the foot Measure abdominal girth Ask pt to extend the arms Instruct pt to lean forward

Ask the pt to extend the arms Asterixis is irregular flapping movements of the fingers and wrists when the hands and arms are outstretched, with the palms down, wrists bent up, and fingers spread. It is the most common and reliable sign that hepati encephalopathy is developing.

Your patient is post-op day 3 from a cholecystectomy due to cholecystitis and has a T-Tube. Which finding during your assessment of the T-Tube requires immediate nursing intervention?* A. The drainage from the T-Tube is yellowish/green in color. B. There is approximately 750 cc of drainage within the past 24 hours. C. The drainage bag and tubing is at the patient's waist. D. The patient is in the Semi-Fowler's position.

B

An older adult pt presents to the ED with a 4-day history of cough, SOB, pain on inspiration, and dyspnea. The pt never had a pneumococcal vaccine. The chest x-ray shows density in bilateral lung bases. The pt has wheezing upon auscultation of both lungs. Would a bronchodilator be beneficial for this pt? A. It would not be beneficial for this pt. B. It would help decrease the bronchospasm. C. It would clear up the density in the pts lung bases. D. The pt would have a decrease in the pain on inspiration.

B A bronchodilator would would open up the airways and help decrease bronchospasm , so it would be beneficial for this pt. It would decrease dyspnea and feelings of shortness of breath.

The pt with advanced cirrhosis asks why his abdomen is so swollen. The nurse's best response is based on the knowledge that a. a lack of clotting factors promotes the collection of blood in the abdominal cavity b. portal hypertension and hypoalbuminemia cause fluid shift into the peritoneal space. c. decreased peristalsis in the GI tract contributes to gas formation and distention of the bowel d. bile salts in the blood irritate the peritoneal membranes, causing edema and pocketing of fluid.

B Ascites is accumulation of serious fluid in peritoneal cavity. With portal hypertension, protein shifts from the blood into the lymph. When the lymph system is unable to carry excess, it leaks thru the liver into the peritoneal cavity. osmotic pressure of the proteins pulls additional fluid into cavity. Second mechanism of ascites if hypoalbuminemia from the liver unable to synthesize albumin, resulting in decreased colloidal oncotic pressure.

INH treatment is associated with the development of peripheral neuropathies. Which of the following interventions would the nurse teach the client to help prevent this complication? A. Adhere to a low cholesterol diet B. Supplement the diet with pyridoxine (vitamin B6) C. Get extra rest D. Avoid excessive sun exposure

B INH competes with the available vitamin B6 in the body and leaves the client at risk for development of neuropathies related to vitamin deficiency. Supplemental vitamin B6 is routinely prescribed.

A pt has been told she has NAFLD. The nursing teaching plan should include a. having genetic testing done b. recommend a heart healthy diet c. the necessity to reduce weight rapidly d. avoiding alcohol until liver enzymes return to normal

B NAFLD can progress to cirrhosis. NO definitive treatment; therapy directed at reducing risk like diabetes, body weight, and harmful medications.

Which symptom of pneumonia may present differently in the older adult pt than in the younger adult pt? A. Crackles on auscultation B. Fever C. Headache D. Wheezing

B Older adults may not have fever and may have a lower-than-normal temperature with pneumonia. The other answers may be seen in all age groups of pneumonia pts

The nurse is caring for a 55-yr-old man patient with acute pancreatitis resulting from gallstones. Which clinical manifestation would the nurse expect? A) Hematochezia B) Left upper abdominal pain C) Ascites and peripheral edema D) Temperature over 102 F

B Rationale: Abdominal pain (usually in the left upper quadrant) is the predominant manifestation of acute pancreatitis. Other manifestations of acute pancreatitis include nausea and vomiting, low-grade fever, leukocytosis, hypotension, tachycardia, and jaundice. Abdominal tenderness with muscle guarding is common. Bowel sounds may be decreased or absent. Ileus may occur and causes marked abdominal distention. Areas of cyanosis or greenish to yellow-brown discoloration of the abdominal wall may occur. Other areas of ecchymoses are the flanks (Grey Turner's spots or sign, a bluish flank discoloration) and the periumbilical area (Cullen's sign, a bluish periumbilical discoloration).

A nurse is providing discharge teaching for a client who has chronic hepatitis C. Which of the following statements indicates an understanding of the teaching? A. "I will avoid alcohol until I'm no longer contagious." B. "I will avoid medications that contain acetaminophen." C. "I will decrease my intake of calories." D. "I will need treatment for 3 months."

B. "I will avoid medications that contain acetaminophen." A client who has hepatitis C should avoid medications that contain acetaminophen, which can cause additional liver damage.

A client has developed acute pancreatitis after also developing gallstones. Which is the highest priority nursing instruction for this client to avoid further attacks of pancreatitis? A. "You may need a surgical consult for removal of your gallbladder." B. "See your health care provider (HCP) immediately when experiencing symptoms of a gallbladder attack." C. "If you have a gallbladder attack and pain does not resolve within a few days, call your health care provider." D. "You'll need to drastically modify your alcohol intake."

B. "See your health care provider (HCP) immediately when experiencing symptoms of a gallbladder attack."

A nurse is caring for a client who has colorectal cancer and is receiving chemo. The client asks the nurse why his blood is drawn for a carcinoembryonic antigen (CEA) level. Which of the following responses should the nurse make? A."The CEA determines the current stage of your colon cancer." B. "The CEA determines the efficacy of your chemotherapy." C. "The CEA determines if the neutrophil count is below the expected reference range." D. "The CEA determines if you are experiencing occult bleeding from the gastrointestinal tract."

B. "The CEA determines the efficacy of your chemotherapy." A provider uses the CEA level to determine the efficacy of the chemotherapy. The client's CEA levels will decrease if the chemotherapy is effective.

A client has just been diagnosed with pancreatic cancer. The client's upset spouse tells the nurse that they have recently moved to the area, have no close relatives, and are not yet affiliated with a church. What is the nurse's best response? A. "Maybe you should find a support group to join." B. "Would you like me to contact the hospital chaplain for you?" C. "Do you want me to try to find a therapist for you?" D. "Do you have any friends whom you want me to call?"

B. "Would you like me to contact the hospital chaplain for you?"

A nurse is completing preoperative teaching for a client who is scheduled for a laparoscopic cholecystectomy. Which of the following should be included in the teaching? A. "The scope will be passed through your rectum." B. "You might have shoulder pain after surgery." C. "You will have a Jackson-Pratt drain in place after surgery." D. "You should limit how often you walk for 1 to 2 weeks."

B. "You might have shoulder pain after surgery."

When preparing a client to undergo paracentesis, which action is necessary to reduce potential injury as a result of the procedure? A. Encourage the client to take deep breaths and cough B. Ask the client to void prior to the procedure C. Position the client with the head of the bed flat D. Assist the physician to insert a trocar catheter into the abdomen

B. Ask the client to void prior to the procedure

A nurse is assessing a client who has peritonitis. Which of the following findings should the nurse expect? A. Bloody diarrhea B. Board-like abdomen C. Periumbilical cyanosis D. Increased bowel sounds

B. Board-like abdomen A board-like, distended abdomen, accompanied by extreme pain and tenderness, is an expected finding in a client who has peritonitis.

A nurse is assessing a client who has advanced cirrhosis. The nurse should identify which of the following findings as indicators of hepatic encephalopathy? (Select all that apply.) A. Anorexia B. Change in orientation C. Asterixis D. Ascites E. Fetor hepaticus

B. Change in orientation C. Asterixis E. Fetor hepaticus

The nurse is caring for a patient with unstable angina whose cardiac monitor shows ventricular tachycardia. Which action is appropriate to implement first? A. Defibrillate the patient at 200 joules. B. Check the patient for a pulse. C. Cardiovert the patient at 50 joules. D. Give the patient IV lidocaine.

B. Check the patient for a pulse.

A nurse is providing discharge teaching to a client who is postoperative following open cholecystectomy with T-tube placement. Which of the following instructions should the nurse include in the teaching? (Select all that apply.) A. Take baths rather than showers. B. Clamps T-tube for 1 hr before and after meals. C. Keep the drainage system above the level of the abdomen. D. Expect to have the T-tube removed 3 days postoperatively. E. Report brown-green drainage to the provider.

B. Clamps T-tube for 1 hr before and after meals. C. Keep the drainage system above the level of the abdomen.

The RN has just received the change-of-shift report for the medical unit. Which client should the RN see first? A. Client with ascites who had a paracentesis 2 hours ago and is reporting a headache B. Client with portal-systemic encephalopathy (PSE) who has become increasingly difficult to arouse C. Client with hepatic cirrhosis and jaundice who has hemoglobin of 10.9 g/dL (109 mmol/l) and thrombocytopenia D. Client with hepatitis A who has elevated alanine aminotransferase (ALT) and aspartate aminotransferase (AST)

B. Client with portal-systemic encephalopathy (PSE) who has become increasingly difficult to arouse

The nurse is caring for clients in the outclient clinic. Which of these phone calls would the nurse return first? A. Client with hepatitis A reporting severe and ongoing itching B. Client with severe ascites who has a temperature of 101.4°F (38°C) C. Client with cirrhosis who has had a 3-pound (1.4 kg) weight gain over 2 days D. Client with esophageal varices and mild right upper quadrant pain

B. Client with severe ascites who has a temperature of 101.4°F (38°C)

When providing dietary teaching to a client with hepatitis, what practice does the nurse recommend? A. Having a larger meal early in the morning B. Consuming increased carbohydrates and moderate protein C. Restricting fluids to 1500 mL/day D. Limiting alcoholic beverages to once weekly

B. Consuming increased carbohydrates and moderate protein

A nurse is providing dietary teaching for a client who is postoperative following a gastrectomy. Which of the following foods should the nurse encourage the client to include in her diet to prevent dumping syndrome? A. Ice cream B. Eggs C. Grape juice D. Honey

B. Eggs The nurse should instruct the client to increase intake of protein-containing foods, such as eggs, to decrease the risk for manifestations of dumping syndrome. The client should eat some form of protein at each meal.

Which activity by the nurse will best relieve symptoms associated with ascites? A. Administering oxygen B. Elevating the head of the bed C. Monitoring serum albumin levels D. Administering intravenous fluids

B. Elevating the head of the bed

Which risk factors are known to contribute to atrial fibrillation? (Select all that apply.) A. Use of beta-adrenergic blockers B. Excessive alcohol use C. Advancing age D. High blood pressure E. Palpitations

B. Excessive alcohol use C. Advancing age D. High blood pressure

A nurse is providing teaching for a client who has cirrhosis and a new prescription for lactulose. THe nurse should include which of the following instructions in the teaching. A. Notify provider if bloating occurs B. Expect to have 2-3 soft stools per day C. Restrict carbohydrates in the diet D. Limit oral fluid intake to 1000 mL per day of clear fluids

B. Expect ot have 2-3 soft stools per day The purpose of administering lactulose is to promote excretion of ammonia in stool. the nurse should instruct the client to take he medication every day and inform the client that 2-3 bowel movements everyday is the treatment goal.

a nurse is providing discharge teaching for a client who has a new prescription for medications to treat peptic ulcer disease. The nurse should identify that which of the gollowing medications inhibits gastric acid secretion? A. Calcium carbonate B. Famotidine C. Aluminum hydroxide D. Sucralfate

B. Famotidine The nurse should inform the client that famotidine is an H2-receptor antagonist that is prescribed for the treatment of peptic ulcer disease to inhibit the secretion of gastric acid.

The nurse is caring for a patient with acute coronary syndrome (ACS) and atrial fibrillation who has a new prescription for metoprolol (Toprol). Which monitoring is essential when administering the medication? A. ST segment B. Heart rate C. Troponin D. Myoglobin

B. Heart rate

A nurse is reviewing the laboratory results of a client who has acute pancreatitis. Which of the following findings should the nurse expect? A. Blood glucose 110 mg/dL B. Increased serum amylase C. WBC 9,000/mm3 D. Decreased bilirubin

B. Increased serum amylase Serum amylase levels are increased in a client who has acute pancreatitis due to pancreatic cell injury.

A client is scheduled to undergo a liver transplantation. Which nursing intervention is most likely to prevent the complications of bile leakage and abscess formation? A. Preventing hypotension B. Keeping the T-tube in a dependent position C. Administering antibiotic vaccinations D. Administering immune-suppressant drugs

B. Keeping the T-tube in a dependent position

A nurse is reviewing the prescriptions for a client who has Campylobacter enteritis. Which of the following prescriptions should the nurse clarify with the provider? A. 0.45% sodium chloride IV B. Magnesium hydroxide C. Ciprofloxacin D. Potassium

B. Magnesium hydroxide Nausea, vomiting, and diarrhea are manifestations of enteritis. The nurse should clarify a prescription for magnesium hydroxide, also known as milk of magnesia, with the provider. This medication increases gastrointestinal motility, which can increase the client's risk for an electrolyte imbalance and contribute to dehydration.

When providing community education, the nurse emphasizes that which group needs to receive immunization for hepatitis B? A. Clients who work with shellfish B. Men who engage in sex with men C. Clients traveling to a third-world country D. Clients with elevations of aspartate aminotransferase and alanine aminotransferase

B. Men who engage in sex with men

The nurse expects that which client will be discharged to the home environment first? A. Older obese adult who has had a laparoscopic cholecystectomy B. Middle-aged thin adult who has had a laparoscopic cholecystectomy C. Middle-aged thin adult with a heart murmur who has had a traditional cholecystectomy D. Older obese adult with chronic obstructive pulmonary disease (COPD) who has had a traditional cholecystectomy

B. Middle-aged thin adult who has had a laparoscopic cholecystectomy

The nurse suspects that which client is at highest risk for developing gallstones? A. Obese male with chronic obstructive pulmonary disease B. Obese female receiving hormone replacement therapy C. Thin male with a history of coronary artery bypass grafting D. Thin female who has recently given birth

B. Obese female receiving hormone replacement therapy

A nurse is preparing to administer pancrelipase to a client who has pancreatitis. Which of the following actions should the nurse take? A. Instruct the client to chew the medication before swallowing. B. Offer a glass of water following medication administration. C. Administer the medication 30 min before meals. D. Sprinkle the contents on peanut butter.

B. Offer a glass of water following medication administration.

A nurse on a medical unit is caring for a patient who apirated gastric contents prior to admission. The nurse administers 100% oxygen by nonbreather mask after the patient reports severe dyspnea. Which of the following findings is a clinical manifestation of acute respiratory distress syndrome (ARDS)? A. Tympanic temperature of 38 C (100.4 F) B. PaO2 50 mm Hg C. Rhonchi D. Hypopnea

B. PaO2 50 mm Hg - The patient who has manifestations of ARDS has a low PaO2 level even with the administration of oxygen. Hypoxemia after treatment with oxygen is a manifestation of ARDS. -The client who has ARDS will have clear breath sounds because edema occurs in the interstitial spaces and not in the airway. -The client who has ARDS will manifest hyperpnea

The nurse receives a report that a patient with a pacemaker has experienced loss of capture. Which situation is consistent with this? A. The pacemaker spike falls on the T wave. B. Pacemaker spikes are noted, but no P wave or QRS complex follows. C. The heart rate is 42 beats/min, and no pacemaker spikes are seen on the rhythm strip. D. The patient demonstrates hiccups.

B. Pacemaker spikes are noted, but no P wave or QRS complex follows.

A nurse is caring for a client who has a new diagnosis of hepatitis C. Which of the following laboratory findings should the nurse expect? A. Presence of immunoglobulin G antibodies (IgG) B. Presence of enzyme immunoassay (EIA) C. Aspartate aminotransferase (AST) 35 units/L D. Alanine aminotransferase (ALT) 15 IU/L

B. Presence of enzyme immunoassay (EIA)

A nurse is providing instructions about pursed-lip breathing for a patient who has COPD with emphysema. The nurse should explain that this breathing technique accomplishes which of the following? A. Increases oxygen intake B. Promotes carbon dioxide elimination C. Uses the intercostal muscles D. Strengthens the diaphram

B. Promotes carbon dioxide elimination - A patient who has COPD with emphysema should use pursed-lip breathing when experiencing dyspnea. This is one of the simplest ways to control dyspnea. It slows the patient's pace of breathing, making each breath more effective. Pursed-lip breathing releases trapped air in the lungs and prolongs exhalation to slow the breathing rate. This improved breathing pattern moves carbon dioxide out of the lungs more efficiently.

The nurse administers amiodarone (Cordarone) to a patient with ventricular tachycardia. Which monitoring by the nurse is necessary with this drug? (Select all that apply.) A. Respiratory rate B. QT interval C. Heart rate D. Heart rhythm E. Urine output

B. QT interval C. Heart rate D. Heart rhythm

A nurse is caring for an older adult client who has chronic obstructive pulmonary disease with pneumonia. The nurse should monitor the client for which of the following acid-base imbalances? A. Respiratory alkalosis B. Respiratory acidosis C. Metabolic alkalosis D. Metabolic acidosis

B. Respiratory acidosis - Respiratory acidosis is a common complication of COPD. This complication occurs because patients who have COPD are unable to exhale carbon dioxide due to a loss of elastic recoil in the lungs.

A nurse is developing a teaching plan for a client about preventing acute asthma attacks. Which of the following points should the nurse plan to discuss first? A. How to eliminate environmental triggers that precipitate attacks B. The client's perception of the disease process and what might have triggered past attacks C. The client's medication regimen D. Manifestations of respiratory infections

B. The client's perception of the disease process and what might have triggered past attacks - The nurse should apply the nursing process priority-setting framework. The nurse can use the nursing process to plan client care and prioritize nursing actions. Each step of the nursing process builds on the previous step, beginning with assessment. Before the nurse can formulate a plan of action, implement a nursing intervention, or notify a provider of a change in the client's status, the nurse must first collect adequate data from the client. Assessing the client will provide the nurse with the knowledge to make an appropriate decision. Therefore, the first step the nurse should take is to assess the client's current knowledge.

A nurse is teaching about daily chest physiotherapy with a client who has cystic fibrosis. The nurse should instruct the client that which of the following is the purpose of the treatments? A. To encourage deep breaths B. To mobilize secretions in the airways C. To dilate the bronchioles D. To stimulate the cough reflex

B. To mobilize secretions in the airways The purpose of chest physiotherapy is to loosen the patient's secretions and promote drainage of secretions from the lungs. Chest physiotherapy includes percussion, vibration, and promotion of drainage by gravity.

The condition of the pt who has cirrhosis of the liver has deteriorated. Which diagnostic study would help determine if the pt has developed liver cancer? a. serum a-fetoprotein level b. ventilation/perfusion scan c. hepatic structure ultasound d. abdominal girth measurement

C

Your patient is unable to have a cholecystectomy for the treatment of cholecystitis. Therefore, a cholecystostomy tube is placed to help treat the condition. Which statement about a cholecystostomy (C-Tube) is TRUE?* A. The C-Tube is placed in the cystic duct of the gallbladder and helps drain infected bile from the gallbladder. B. Gallstones regularly drain out of the C-Tube, therefore, the nurse should flush the tube regularly to ensure patency. C. The C-Tube is placed through the abdominal wall and directly into the gallbladder where it will drain infected bile from the gallbladder. D. The tubing and drainage bag of the C-Tube should always be level with the insertion site to ensure the tube is draining properly.

C

The client diagnosed with acute pancreatitis is in pain. Which position should the nurse assist the client to assume to help decrease the pain? A. Recommend lying in the prone position with legs extended B. Maintain a tripod position over the bedside table C. Plance in side-lying position with knees flexed D. Encourage a supine position with a pillow under the knees

C Rationale: The fetal position deceases pain caused by the stretching of the peritoneum as a result of edema. The pancreas is located abdomen. Anything that causes the abdomen to be stretched will increase pain.

The nurse provides discharge instructions for a 64 y.o. women with ascites and peripheral edema related to cirrhosis. Which statement, if made by the pt, indicates teaching WAS effective? a. it is safe to take acetaminophen up to four times a day for pain b. lactulose (cephulac) should be taken everyday to prevent constipation c. herbs and other spices should be used to season my foods instead of salt d. i will eat foods high in potassium while taking spironolactone (aldactone)

C low sodium diet is indicated for pt with ascites and edema related to cirrhosis

The pneumonia pt asks the nurse why she needs to draw labs to check his electrolytes. The nurses correct response is A. To monitor for possible acidosis B. To check for elevated WBCs C. To evaluate the sodium level for possible hypernatremia D. To check for possible septicemia

C Because of dehydration from fever, may be hypernatemic/ dehydrated Electrolytes don't show acidosis, that is ABGS It is important to check the WBC count but its not an electrolyte A blood culture would need to be done for septicemia

The nurse identifies the priority nursing dx for a pneumonia pt to be ineffective airway clearance related to fatigue, CP, excessive secretions and muscle weakness. To correct the problem the nurse will implement which intervention A. administer oxygen to prevent hypoxemia and atelectasis B. push fluids to greater than 3000 mL/day to ensure adequate hydration C. administer bronchodilator therapy in a timely manner to decrease bronchospasms D. maintain semi-fowlers position to facilitate breathing and prevent further fatigue

C bronchodilator

A pt is admitted to the hospital with bronchopneumonia. The nurse knows that this pt has pneumonia that A. has only affected a certain lobe of the lung B. has affected bilateral lower lobes C. is scattered throughout the lung, with affected patches throughout. D. will cause aspiration, so pt should be monitored

C bronchopneumonia is scattered affected areas throughout multiple lobes of the lungs A. = a definition of lobar pneumonia

A nurse is providing discharge teaching for an older adult client who has mild diverticulitis. Which of the following statements by the client indicates an understanding of the teaching? A. "I may experience right lower quadrant pain." B. "I will remain active by working in my garden every day." C. "I should eat foods that are low in fiber." D. "I will use a mild laxative every day."

C. "I should eat foods that are low in fiber." The nurse should instruct the client who has diverticulitis to follow a low-fiber diet. When the inflammation subsides, the client should consume foods that are high in fiber.

A nurse is providing dietary teaching for a client who has a new diagnosis of celiac disease. Which of the following statements by the client indicates an understanding of the teaching? A. "I can return to my regular diet when I am free of symptoms." B. "I will need to avoid taking vitamin supplements while on this diet." C. "I will eat beans to ensure I get enough fiber in my diet." D. "I need to avoid drinking liquids with my meals while on this diet."

C. "I will eat beans to ensure I get enough fiber in my diet." Clients who have celiac disease must maintain a gluten-free diet which eliminates fiber-rich whole wheat products. Clients should eat beans, nuts, fruits, and vegetables to ensure an adequate intake of fiber.

A client has been discharged to home after being hospitalized with an acute episode of pancreatitis. The client, who is an alcoholic, is unwilling to participate in Alcoholics Anonymous (AA), and the client's spouse expresses frustration to the home health nurse regarding the client's refusal. What is the nurse's best response? A. "Your spouse will sign up for the meetings only when he is ready to deal with his problem." B. "Keep mentioning the AA meetings to your spouse on a regular basis." C. "I'll get you some information on the support group Al-Anon." D. "Tell me more about your frustration with your spouse's refusal to participate in AA."

C. "I'll get you some information on the support group Al-Anon."

When assessing a client for hepatic cancer, the nurse anticipates finding an elevation in which laboratory test result? A. Hemoglobin and hematocrit B. Leukocytes C. Alpha-fetoprotein D. Serum albumin

C. Alpha-fetoprotein

A patient with atrial fibrillation (AF) with rapid ventricular response has received medication to slow the ventricular rate. The pulse is now 88 beats/min. For which additional therapy does the nurse plan? A. Synchronized cardioversion B. Electrophysiology studies (EPS) C. Anticoagulation D. Radiofrequency ablation therapy

C. Anticoagulation

A nurse is assessing a client who has cirrhosis. Which of the following findings is a priority for the nurse to report to the provider? A. Spider angiomas B. Peripheral edema C. Bloody stools D. Jaundice

C. Bloody stools The greatest risk to the client who has cirrhosis of the liver is hemorrhagic shock due to bleeding in the esophageal varices. Therefore, bloody stools is the priority finding to report to the provider.

The nursing team consists of an RN, an LPN/LVN, and a nursing assistant. Which client should be assigned to the RN? A. Client who is taking lactulose and has diarrhea B. Client with hepatitis C who requires a dressing change C. Client with end-stage cirrhosis who needs teaching about a low-sodium diet D. Obtunded client with alcoholic encephalopathy who needs a blood draw

C. Client with end-stage cirrhosis who needs teaching about a low-sodium diet

A patient is admitted to the emergency department following a motorcycle crash. The nurse notes a crackling sensation upon palpation on the right side of the patient's chest. After notifying the provider, the nurse should document the finding as which of the following? A. Friction rub B. Crackles C. Crepitus D. Tactile fremitus

C. Crepitus Crepitus, also called subcutaneous emphysema, is a coarse crackling sensation that the nurse can feel when palpating the skin surface over the patient's chest. Crepitus indicates an air leak into the subcutaneous tissue, which is often a clinical manifestation of a pneumothorax.

A nurse is providing discharge teaching for a client following an ileostomy. The nurse should instruct the client to report which of the following findings to the provider? A. Intolerance to high-fiber foods B. Liquid ileostomy output C. Dark purple stoma D. Sensation of burning during bowel elimination

C. Dark purple stoma The nurse should instruct the client to contact the provider if the stoma is a dark purple color, which is an indication of bowel ischemia.

A nurse in a clinic is reviewing the laboratory reports of a client who has suspected cholelithiasis. Which of the following is an expected finding? A. Serum amylase 80 units/L B. WBC 9,000/mm3 C. Direct bilirubin 2.1 mg/dL D. Alkaline phosphatase 25 units/L

C. Direct bilirubin 2.1 mg/dL

A nurse is teaching a client how to prepare for a colonoscopy. Which of the following instructions should the nurse include in the teaching? A. Begin drinking the oral liquid preparation for bowel cleansing on the morning of the procedure. B. Drink full liquids for breakfast the day of the procedure, and then take nothing by mouth for 2 hr prior to the procedure. C. Drink clear liquids for 24 hr prior to the procedure, and then take nothing by mouth for 6 hr before the procedure. D. Drink the oral liquid preparation for bowel cleansing slowly.

C. Drink clear liquids for 24 hr prior to the procedure, and then take nothing by mouth for 6 hr before the procedure. The nurse should instruct the client to drink clear liquids for 24 hr prior to the colonoscopy to promote adequate bowel cleansing. Maintaining NPO status for 4 to 6 hr prior to the colonoscopy preserves the bowel's cleansed state.

The nurse suspects that a client may have acute pancreatitis as evidenced by which group of laboratory results? A. Deceased calcium, elevated amylase, decreased magnesium B. Elevated bilirubin, elevated alkaline phosphatase C. Elevated lipase, elevated white blood cell (WBC) count, elevated glucose D. Decreased blood urea nitrogen (BUN), elevated calcium, elevated magnesium

C. Elevated lipase, elevated white blood cell (WBC) count, elevated glucose

A nurse is assessing a client who has upper GI bleeding. Which of the following findings should the nurse expect? A. Bradycardia B. Bounding peripheral pulses C. Hypotension D. Increased hematocrit levels

C. Hypotension A client who has upper gastrointestinal bleeding is at risk for hemorrhagic shock. Hypotension is a manifestation of hemorrhagic shock.

A nurse is admitting a client who has acute pancreatitis. Which of the following actions should the nurse take first? A. Insert a nasogastric tube for the client. B. Administer ceftazidime to the client. C. Identify the client's current level of pain. D. Instruct the client to remain NPO.

C. Identify the client's current level of pain. The first action the nurse should take when using the nursing process is to assess the client. Clients who have acute pancreatitis often have severe abdominal pain. By assessing the client's level of pain, the nurse can identify the need for and implement interventions to alleviate the client's pain.

A nurse is assessing a client who has pancreatitis. Which of the following actions should the nurse take to assess the presence of Cullen's sign? A. Tap lightly at the costovertebral margin on the client's back. B. Palpate the RLQ. C. Inspect the skin around the umbilicus. D. Auscultation the area below the scapula.

C. Inspect the skin around the umbilicus.

What teaching does the home health nurse give the family of a client with hepatitis C to prevent the spread of the infection? A. The client must not consume alcohol. B. Avoid sharing the bathroom with the client. C. Members of the household must not share toothbrushes. D. Drink only bottled water and avoid ice.

C. Members of the household must not share toothbrushes.

A nurse in a provider's office is assessing a client who states he was recently exposed to tuberculosis. Which of the following findings is a clinical manifestation of pulmonary tuberculosis? A. Pericardial friction rub B. Weight gain C. Night sweats D. Cyanosis of the fingertips

C. Night sweats - Night sweats, fevers, anorexia and weight loss are clinical manifestations of tuberculosis. -A pericardial friction rub is a clinical manifestation of rheumatic carditis. -Cyanosis of the fingertips is a clinical manifestation of Raynaud's disease.

A client with a history of esophageal varices has just been admitted to the emergency department after vomiting a large quantity of blood. Which action does the nurse take first? A. Obtain the charts from the previous admission. B. Listen for bowel sounds in all quadrants. C. Obtain pulse and blood pressure. D. Ask about abdominal pain.

C. Obtain pulse and blood pressure.

A nurse is preparing to assist a provider to withdraw arterial blood from a patient's radial artery for measurement of ABG. Which of the following actions should the nurse plan to take? A. Hyperventilate the patient with 100% oxygen prior to obtaining the specimen. B. Apply ice to the site after obtaining the specimen. C. Perform an Allen's test prior to obtaining the specimen. D. Release pressure applied to the puncture site 1 minute after the needle is withdrawn.

C. Perform an Allen's test prior to obtaining the specimen. - The nurse should ensure that circulation to the hand is adequate from the ulnar artery in case the radial artery is injured from the blood draw. The most common site for withdrawal of arterial blood gases is the radial artery.

A patient admitted after using crack cocaine develops ventricular fibrillation. After determining unresponsiveness, which action does the nurse take next? A. Prepare for defibrillation. B. Establish IV access. C. Place an oral airway and ventilate. D. Start cardiopulmonary resuscitation (CPR).

C. Place an oral airway and ventilate.

Which problem for a client with cirrhosis takes priority? A. Insufficient knowledge related to the prognosis of the disease process B. Discomfort related to the progression of the disease process C. Potential for injury related to hemorrhage D. Inadequate nutrition related to an inability to tolerate usual dietary intake

C. Potential for injury related to hemorrhage

The nurse is assessing a client who has recurrent attacks of pancreatitis and is concerned about possible alcohol abuse as an underlying cause of these attacks. To elicit this information, what will the nurse do initially? A. Ask the client about binge drinking. B. Question the client whether drinking increases on weekends. C. Provide privacy and use the CAGE questionnaire (Cut down, Annoyed by criticism, Guilt about drinking, and Eye-opener drinks) D. Ask the client's spouse to describe the client's drinking

C. Provide privacy and use the CAGE questionnaire (Cut down, Annoyed by criticism, Guilt about drinking, and Eye-opener drinks)

Which intervention provides safety during cardioversion? A. Setting the defibrillator at 220 joules B. Obtaining informed consent C. Setting the defibrillator to the synchronized mode D. Removing oxygen

C. Setting the defibrillator to the synchronized mode

A patient's rhythm strip shows a heart rate of 116 beats/min, one P wave occurring before each QRS complex, a PR interval measuring 0.16 second, and a QRS complex measuring 0.08 second. How does the nurse interpret this rhythm strip? A. Normal sinus rhythm B. Sinus bradycardia C. Sinus tachycardia D. Sinus rhythm with premature ventricular contractions

C. Sinus tachycardia

A nurse is planning care for a patient following placement of a chest tube 1 hour ago. Which of the following actions should the nurse include in the plan of care? A. Clamp the chest tube if there is continuously bubbling in the water seal chamber B. Keep the chest tube drainage system at the level of the right atrium C. Tape all of the connections between the chest tube and the drainage system D. Empty the collection chamber and record the amount of drainage every 8 hours

C. Tape all of the connections between the chest tube and the drainage system - The nurse should tape all of the connections to ensure that the system is airtight and prevent the chest tubing from accidentally disconnecting.

When assessing a client with hepatitis B, the nurse anticipates which assessment findings? Select all that apply. A. Recent influenza infection B. Brown stool C. Tea-colored urine D. Right upper quadrant tenderness E. Itching

C. Tea-colored urine D. Right upper quadrant tenderness E. Itching

The professional nurse is supervising a nursing student performing a 12-lead electrocardiogram (ECG). Under which circumstance does the nurse correct the student? A. The patient is semi-recumbent in bed. B. Chest leads are placed as for the previous ECG. C. The patient is instructed to breathe deeply through the mouth. D. The patient is instructed to lie still.

C. The patient is instructed to breathe deeply through the mouth.

The nurse administers lactulose (Evalose) to a client with cirrhosis for which purpose? A. To aid in digestion of dairy products B. To reduce portal pressure C. To promote gastrointestinal (GI) excretion of ammonia D. To reduce the risk of GI bleeding

C. To promote gastrointestinal (GI) excretion of ammonia

The nurse is teaching a client with gallbladder disease about diet modification. Which meal does the nurse suggest to the client? A. Steak and French fries B. Fried chicken and mashed potatoes C. Turkey sandwich on wheat bread D. Sausage and scrambled eggs

C. Turkey sandwich on wheat bread

A nurse is providing discharge teaching for a client who has a new colostomy and is concerned about flatus and odor. Which of the following foods should the nurse recommend to the client? A. Eggs B. Fish. C. Yogurt D. Broccoli

C. Yogurt The nurse should recommend yogurt, cracker and toast, which can prevent flatus and odor.

What is the difference between acute and chronic pancreatitis?

Chronic occurs over a long period of time, is degenerative, and not curable. Acute occurs quickly, is not reoccurring, and can be cured. Acute is more aggressive than chronic.

Hypermagnesemia complications

Complications. Severe hypermagnesemia (levels greater than 12 mmol/dL) can lead to cardiovascular complications (hypotension and arrhythmias) and neurological disorder (confusion and lethargy). Higher values of serum magnesium (exceeding 15 mg/dL) can induce cardiac arrest and coma

Who should not take the pneumonia and flu vaccines?

Contraindications for use (of CAIV-T influenza vaccine) include anaphylactic reactions to eggs, a history of Guillain-Barr syndrome, patients aged <18 years on long-term aspirin therapy, pregnancy during the first trimester, and various states of immunosuppression.

How are crohn's and ulcerative colitis different?

Crohn's disease can occur anywhere in the bowel and there is an autoimmune component. Fistulas occur in the intestine. It is not curable. Ulcerative colitis only happens in the lower part of the bowel and causes the bowel to become inflammed. It is curable through surgical removal of the effected part of the bowel.

When planning care for a pt with cirrhosis, the nurse will give highest priority to which nursing diagnosis? a. impaired skin integrity related to edema, ascites, and pruritis b. imbalanced nutrition: less than body requirements related to anorexia c. excess fluid volume related to portal hypertension and hyperaldosteronism d. ineffective breathing pattern related to pressure on diaphragm and reduced lung volume

D airway and breathing are always highest priority.

The health care provider orders lactulose for a patient with hepatic encephalopathy. The nurse will monitor for effectiveness of this medication for this pt by assessing what? a. relief of constipation b. relief of ab pain c. decreased liver enzymes d. decreased ammonia levels

D hepatic encephalopathy is associated with elevated ammonia levels. Lactulose traps ammonia in the intestinal tract. It's laxative effect then expels ammonia from the colon, resulting in decreased ammonia levels, correcting hepatic encephalopathy.

A client's ABG analysis reveals a pH of 7.18, PaCO2 of 72 mm Hg, PaO2 of 77 mm Hg, and HCO3- of 24 mEq/L. What do these values indicate? A. Metabolic acidosis B. Respiratory alkalosis C. Metabolic alkalosis D. Respiratory acidosis

D Respiratory Acidosis

The nurse is reviewing the lab results for an older adult pneumonia pt. The lab value frequently seen in pneumonia pts that may not be seen in this pt is A. RBC 4.0 - 5.0 B. Hgb 12 - 16 C. Hct 36 - 48 D. WBC 12 - 18

D WBC

The nurse is reviewing a pneumonia pts lab results. What does she expect to see A. decreased Hgb B. increased RBCs C. decreased neutrophils D. increased WBCs

D incr WBCs

The client is diagnosed with acute pancreatitis. What health-care provider's admitting order should the nurse question? A. Bedrest with bathroom privileges B. Initiate IV therapy of D5W at 125 mL/hr C. Weight client daily D. Low fat, low car diet

D Rationale: Bedrest decreases the metabolic rate. The client should be NPO to rest the pancreas to decrease the auto digestion of the pancreas. Since the client is NPO IV therapy is appropriate. Weight changes will happen as a result of diet and IV fluids therefore daily weights is appropriate.

Which statement by a client with cirrhosis indicates that further instruction is needed about the disease? A. "Cirrhosis is a chronic disease that has scarred my liver." B. "The scars on my liver create problems with blood circulation." C. "Because of the scars on my liver, blood clotting and blood pressure are affected." D. "My liver is scarred, but the cells can regenerate themselves and repair the damage."

D. "My liver is scarred, but the cells can regenerate themselves and repair the damage."

The nurse is caring for a client recently diagnosed with type 1 diabetes mellitus who has had an episode of acute pancreatitis. The client asks the nurse how he developed diabetes when the disease does not run in the family. What is the nurse's best response? A. "The diabetes could be related to your obesity." B. "Look online for general information about diabetes." C. "Do you consume alcohol on a frequent basis?" D. "Type 1 diabetes can occur when the pancreas is affected or destroyed by disease."

D. "Type 1 diabetes can occur when the pancreas is affected or destroyed by disease."

In teaching patients at risk for bradydysrhythmias, what information does the nurse include? A. "Avoid potassium-containing foods." B. "Stop smoking and avoid caffeine." C. "Take nitroglycerin for a slow heartbeat." D. "Use a stool softener."

D. "Use a stool softener."

A nurse is reviewing the laboratory results of a client who has hepatic cirrhosis. Which of the following laboratory findings should the nurse report to the provider? A. Albumin 4.0 g/dL B. INR 1.0 C. Direct bilirubin 0.5 mg/dL D. Ammonia 180 mcg/dL

D. Ammonia 180 mcg/dL An ammonia level of 180 mcg/dL is above the expected reference range of 10 to 80 mcg/dL. The nurse should report an increased ammonia level because it can indicate portal-systemic encephalopathy.

The nurse asks a client with liver disease to raise the arms to shoulder level and dorsiflex the hands. A few moments later, the hand begins to flap upward and downward. How does the nurse correctly document this in the medical record? A. Positive Babinski's sign B. Hyperreflexia C. Kehr's sign D. Asterixis

D. Asterixis

A nurse us caring for a client who has GERD and a new prescription for metoclopramide. The nurse should plan to monitor for which of the following adverse effects? A. Thrombocytopenia B. Hearing loss C. Hypersalivation D. Ataxia

D. Ataxia The nurse should plan to monitor the client for extrapyramidal symptoms, such as ataxia, and should report any of these findings to the provider.

It is essential that the nurse monitor the client returning from hepatic artery embolization for hepatic cancer for which potential complication? A. Right shoulder pain B. Polyuria C. Bone marrow suppression D. Bleeding

D. Bleeding

The nurse is caring for a patient on a telemetry unit who has a regular heart rhythm and rate of 60 beats/min; a P wave precedes each QRS complex, and the PR interval is 0.20 second. Additional vital signs are as follows: blood pressure 118/68 mm Hg, respiratory rate 16 breaths/min, and temperature 98.8°F (37°C). All of these medications are available on the medication record. What action does the nurse take? A. Administer atropine. B. Administer digoxin. C. Administer clonidine. D. Continue to monitor.

D. Continue to monitor.

A nurse is assessing a client immediately following a paracentesis for the treatment of ascites. Which of the following findings indicates the procedure was effective? A. Presence of a fluid wave B. Increased heart rate C. Equal pre and postprocedure weights D. Decreased SOB

D. Decreased SOB Increased abdominal fluid can limit the expansion of the diaphragm and prevent the client from taking a deep breath. Once excess peritoneal fluid is removed, the diaphragm will expand more freely. The nurse should identify this finding as an indicator the procedure was effective.

A nurse is completing an admission assessment of a client who has pancreatitis. Which of the following findings should the nurse expect? A. Pain in right upper quadrant radiating to right shoulder B. Report of pain being worse when sitting upright C. Pain relieved with defecation D. Epigastric pain radiating to the left shoulder

D. Epigastric pain radiating to the left shoulder

A nurse is completing the admission assessment of a client who has acute pancreatitis. Which of the following findings is the priority to report? A. History of cholelithiasis B. Elevated serum amylase level C. Decrease in bowel sounds upon auscultation D. Hand spasms present when blood pressure is checked

D. Hand spasms present when blood pressure is checked

A nurse is reviewing the laboratory values of a client who has colorectal cancer. Which of the following findings should the nurse expect? A. Negative fecal occult blood test B. Decreased serum carcinoembryonic antigen (CEA) level C. Hematocrit 43% D. Hemoglobin 9.1 g/dL

D. Hemoglobin 9.1g/dL A hemoglobin level of 9.1 g/dL is below the expected reference range. Decreased hemoglobin is an expected finding in a client who has colorectal cancer due to occult intestinal bleeding.

A nurse is providing preoperative teaching to a patient who is to undergo a pneumonectomy. The patient states "I am afraid it will hurt to cough after surgery." Which of the following statements by the nurse is appropriate? A. After the surgeon removes your lung you will not need to cough. B. I'll make sure you get a cough suppressant to keep you from straining the incision when you cough. C. Don't worry. You will have a pump that delivers pain medication as you need it, so you will have very little pain. D. I will show you how to splint your incision while you cough.

D. I will show you how to splint your incision while you cough. The patient who had a pneumonectomy should cough to clear secretions from the remaining lung. The nurse should show how to splint the incision to reduce pain while coughing.

When providing discharge teaching to a client with cirrhosis, it is essential for the nurse to emphasize avoidance of which of these? A. Vitamin K-containing products B. Potassium-sparing diuretics C. Nonabsorbable antibiotics D. Nonsteroidal anti-inflammatory drugs (NSAIDs)

D. Nonsteroidal anti-inflammatory drugs (NSAIDs)

A client with acute cholecystitis is admitted to the medical-surgical unit. Which nursing activity associated with the client's care will be appropriate for the nurse to delegate to unlicensed assistive personnel (UAP)? A. Assessing dietary risk factors for cholecystitis B. Checking for bowel sounds and distention C. Determining precipitating factors for abdominal pain D. Obtaining the admission weight, height, and vital signs

D. Obtaining the admission weight, height, and vital signs

The RN is caring for a client with end-stage liver disease who has ascites. Which action does the RN delegate to unlicensed assistive personnel (UAP)? A. Assessing skin integrity and abdominal distention B. Drawing blood from a central venous line for electrolyte studies C. Evaluating laboratory study results for the presence of hypokalemia D. Placing the client in a semi-Fowler's position

D. Placing the client in a semi-Fowler's position

Which set of assessment findings indicates to the nurse that a client may have acute pancreatitis? A. Absence of jaundice, pain of gradual onset B. Absence of jaundice, pain in right abdominal quadrant C. Presence of jaundice, pain worsening when sitting up D. Presence of jaundice, pain worsening when lying supine

D. Presence of jaundice, pain worsening when lying supine

A client has been placed on enzyme replacement for treatment of chronic pancreatitis. In teaching the client about this therapy, the nurse advises the client not to mix enzyme preparations with foods containing which element? A. Carbohydrates B. High fat C. High fiber D. Protein

D. Protein

A nurse on a medical-surgical unit is admitting a client who has hepatitis B with ascites. Which of the following actions should the nurse include in the plan of care? A. Initiate contact precautions. B. Weigh the client weekly. C. Measure abdominal girth 7.5 cm (3 in) above the umbilicus. D. Provide a high-calorie, high-carbohydrate diet.

D. Provide a high-calorie, high-carbohydrate diet.

A health care worker believes that he may have been exposed to hepatitis A. Which intervention is the highest priority to prevent him from developing the disease? A. Requesting vaccination for hepatitis A B. Using a needleless system in daily work C. Getting the three-part hepatitis B vaccine D. Requesting an injection of immunoglobulin

D. Requesting an injection of immunoglobulin

How does the home care nurse best modify the client's home environment to manage side effects of lactulose (Evalose)? A. Provides small frequent meals for the client B. Suggests taking daily potassium supplements C. Elevates the head of the bed in high-Fowler's position D. Requests a bedside commode for the client

D. Requests a bedside commode for the client

The nurse is attempting to position a client having an acute attack of pancreatitis in the most comfortable position possible. In which position does the nurse place this client? A. Supine, with a pillow supporting the abdomen B. Up in a chair between frequent periods of ambulation C. High-Fowler's position, with pillows used as needed D. Side-lying position, with knees drawn up to the chest

D. Side-lying position, with knees drawn up to the chest

A nurse is caring for a patient who is scheduled for a thoracentesis. In which of the following positions should the nurse place the client? A. Lying flat on the affected side B. Prone with the arms raised over the head C. Supine with the head of the bed elevated D. Sitting while leaning forward over the bedside table

D. Sitting while leaning forward over the bedside table - When preparing a client for a thoracentesis, the nurse should have the client sit on the edged of the bed and lean forward over the bedside table because this position maximizes the space between the client's ribs (intercostal space) and allows for aspiration of accumulated fluid and air.

A nurse is assessing a client who has a duodenal ulcer. Which of the following findings should the nurse expect? A. The client states that the pain is in the upper epigastrium. B. The client is malnourished. C. The client states that ingesting food intensifies the pain. D. The client reports that pain occurs during the night.

D. The client reports that pain occurs during the night. Pain associated with a duodenal ulcer occurs when the stomach is empty, which is typically 1.5 to 3 hr after meals and during the night.

Following paracentesis, during which 2500 mL of fluid was removed, which assessment finding is most important to communicate to the health care provider (HCP)? A. The dressing has a 2-cm area of serous drainage. B. The client's platelet count is 135,000/mm3 (135 × 109/L). C. The client's albumin level is 2.8 g/dL (28 g/L). D. The client's heart rate is 122 beats/min.

D. The client's heart rate is 122 beats/min.

How does the nurse recognize that atropine has produced a positive outcome for the patient with bradycardia? A. The patient states he is dizzy and weak. B. The nurse notes dyspnea. C. The patient has a heart rate of 42 beats/min. D. The monitor shows an increase in heart rate.

D. The monitor shows an increase in heart rate.

A nurse is caring for a client who has a tracheostomy with an inflated cuff in place. Which of the following indicates that the nurse should suction the client's airway secretions? A. The client is unable to speak B. The client's airway secretions were last suctioned 2 hr ago C. The client coughs and expectorates a large mucous plug D. The nurse auscultates coarse crackles in the lung fields

D. The nurse auscultates coarse crackles in the lung fields. - The nurse should auscultate coarse crackles or rhonchi, identify a moist cough, hear or see secretions in the tracheostomy tube, and then suction the client's airway secretions.

A nurse is reviewing a new prescription for ursodiol with a client who has cholelithiasis. Which of the following information should the nurse include in the teaching? A. This medication is used to decrease acute biliary pain. B. This medication requires thyroid function monitoring every 6 months. C. This medication is not recommended for clients who have diabetes mellitus. D. This medication dissolves gallstones gradually over a period of up to 2 years.

D. This medication dissolves gallstones gradually over a period of up to 2 years.

A nurse is preparing to administer cisplatin IV to a client who has lung cancer. The nurse should identify that which of the following findings is an adverse effect of the medication? A. Hallucinations B. Pruritus C. Hand and foot syndrome D. Tinnitus

D. Tinnitus - An adverse effect of cisplatin is ototoxicity, which can cause tinnitus.

The nurse is preparing to instruct a client with chronic pancreatitis who is to begin taking pancrelipase (Cotazym). Which instruction does the nurse include when teaching the client about this medication? A. Administer pancrelipase before taking an antacid. B. Chew tablets before swallowing. C. Take pancrelipase before meals. D. Wipe your lips after taking pancrelipase.

D. Wipe your lips after taking pancrelipase.

​Twenty-two-year-old college student Max Harrington went to the campus health center for a blood test to find out why he was too tired to go to class. Which laboratory value supports the diagnosis of​ influenza?

Decreased WBC -Correct Decreased hematocrit Increased glucose Increased hemoglobin Influenza can cause a​ lower-than-normal WBC count. Changes in the​ CBC, including an increased hemoglobin and decreased​ hematocrit, would not support the diagnosis of influenza. An increased glucose level may indicate​ diabetes, not influenza.

Postoperative care for cholecystectomy

Discharge teaching including s/s of infection, prevention of constipation, low-fat diet, and activity restrictions

The pediatric nurse is talking to a young​ child's grandmother, who cares for the child during the work week. The grandmother​ can't remember whether she gave aspirin or acetaminophen when the child had a fever. What is the best advice the pediatric nurse can give the grandmother for the​ future?

Do not give aspirin to children with a fever.-Correct Write down the medication given as soon as possible. Aspirin or acetaminophen work equally well for fevers. Do not give acetaminophen to children with a fever.

Which ingredients should be avoided in food selection for a client who has irritable bowel syndrome? Select all that apply. Sodium Fructose Sorbitol Dextrose Lactose

Fructose Sorbitol Lactose

A nurse is talking about the symptoms of a patient diagnosed with irritable bowel syndrome (IBS). Which symptom listed indicates a need for further discussion? Sensation of incomplete evacuation More frequent stools at the onset of pain Visible abdominal distension Hard stools at the onset of pain

Hard stools at the onset of pain

A nurse is assessing a client who is postoperative following a gastrectomy. The nurse should identify which of the following findings as an indication of abdominal distention? A. Hiccups B. Hypertension C. Bradycardia D. Chest pain

Hiccups Following surgery, hiccups can be caused by irritation of the phrenic nerve due to abdominal distension. If the hiccups are intractable, the nurse should anticipate a prescription for chlorpromazine because persistent hiccups are distressful to the client and can lead to complications, such as vomiting.

Elevated BUN and creatinine levels indicate

High BUN-to-creatinine ratios occur with sudden (acute) kidney problems, which may be caused by shock or severe dehydration. A very high BUN-to-creatinine ratio may be caused by bleeding in the digestive tract or respiratory tract.

What are some components of the health history for a focused assessment of the client with​ influenza? ​(Select all that​ apply.)

History of influenza vaccination-Correct Presence of productive cough-Correct Chronic diseases-Correct History of influenza Recent colds Some components of the health history are a history of influenza​ vaccination, chronic​ diseases, and presence of a productive cough. A history of influenza or recent colds is not a helpful fact.

With the assistance of a sign language​ interpreter, the clinic nurse is interviewing a deaf client to complete the flu risk health history. The client is coughing and febrile. Which questions are necessary to assess for flu​ risk? ​(Select all that​ apply.)

How long has the client had flu​ symptoms?-Correct What is the client​'s known exposure to the flu​ virus?-Correct What is the client​'s history of influenza​ vaccinations?- Correct What caused the​ client's deafness? How long has the client been​ deaf? Rationale To complete the health​ history, the nurse should ask about known exposure to the flu​ virus; how long the client has had flu​ symptoms; and the history of influenza vaccinations. Questions about the cause and duration of the​ client's deafness are not part of the assessment for flu risk.

As the nurse continues to monitor Kaitlin's progress, which laboratory findings might be observed? Select all that apply. Hypokalemia Hyperglycemia Elevated BUN Decreased creatinine Hypernatremia

Hyperkalemia Elevated BUN Hypernatremia

The nurse is providing nutrition education to Kaitlin, a 35-year-old female client who has a clinical diagnosis of irritable bowel syndrome. Which options should the nurse include with dietary teaching to help manage exacerbation of symptoms? Select all that apply. Maintain a minimum intake of caffeine to prevent caffeine withdrawal Include a consistent level of fiber in the diet (30 to 40 grams per day) to maintain stool bulk Limit intake of carbonated beverages to one can a day Increase daily fluid intake to 6 to 8 glasses of water Ingest gas-forming foods only every other day to prevent cumulative effects

Include a consistent level of fiber in the diet (30-40 grams per day) to maintain stool bulk Increase daily fluid intake to 6-8 glasses of water

Surgical treatment for cholecystitis

Laproscopic cholecystectomy

Decreased WBC Count

Leukopenia, neutropenia, lymphocytopenia, monocytopenia, eosinopenia; Occurs with treatment or disease

What type of diet do patients with IBD need to be on?

Low residue, need refined grain products (white bread, white pasta, white rice, pulp free juice, limit of 2 cups per day of milk products), low fiber foods

What lab values indicate anemia?

Lower than normal hemoglobin levels indicate anemia. The normal hemoglobin range is generally defined as 13.2 to 16.6 grams (g) of hemoglobin per deciliter (dL) of blood for men and 11.6 to 15. g/dL for women.

What does decreased RBCs indicate?

Lower than normal numbers of RBCs may occur with bleeding, bone marrow failure, malnutrition, kidney disease, overhydration, or pregnancy. Several drugs affect the level of RBCs and may make it higher or lower than normal.

Kaitlin expresses concerns that she wants to lose weight. What recommendations should the nurse make to help her attain this goal without exacerbating her IBS? Limit caloric intake to 1,500 calories a day for 1 month Maintain a regular meal schedule Begin an all-liquid diet to stimulate weight loss Include processed frozen foods as meal replacements

Maintain a regular meal schedule

Complications of TB

Miliary TB Pleural Effusion Empyema Acute Pneumonia Other organ involvement

A family of five checks into an urgent care center with symptoms of the flu. Which members of the family are at increased risk of significant​ complications?

Newborn baby girl-Correct Grandmother with COPD-Correct Father with HIV-Correct Mother with anemia ​8-year-old boy Rationale The family members at increased risk of complications are the​ newborn, who fits the category of​ infants; the father with​ HIV, who fits the category of those with a compromised immune​ system; and the grandmother with​ COPD, who fits the category of those with chronic pulmonary disease. Neither the​ 8-year-old boy nor the mother with anemia is at increased risk of complications.

A​ middle-aged client wants to know about the choices of​ over-the-counter drugs for treating flu symptoms. Which nonprescription drugs can the nurse​ suggest? ​(Select all that​ apply.)

Nonsteroidal​ anti-inflammatory drugs-Correct Acetaminophen-Correct Aspirin-Correct Antiemetics Antivirals Rationale Nonprescription drugs for treating flu symptoms include​ aspirin, acetaminophen, and nonsteroidal​ anti-inflammatory drugs. Antiemetics are used to treat nausea and vomiting. Antivirals must be prescribed by a health care provider.

What medications are given for pancreatitis?

Opiod analgesics, anticholinergics (atropine (Atropen) belladonna alkaloids. benztropine mesylate (Cogentin) clidinium. cyclopentolate (Cyclogyl) darifenacin (Enablex) dicylomine. fesoterodine (Toviaz)), histamine blockers (Famotidine (Pepcid AC, Pepcid Oral) Cimetidine (Tagamet, Tagamet HB) Ranitidine (Zantac, Zantac 75, Zantac Efferdose, Zantac injection, and Zantac Syrup) Nizatidine Capsules (Axid AR, Axid Capsules, Nizatidine Capsules), antibiotic therapy

Kaitlin tells the nurse "I know this sounds crazy, but my stomach feels better when I suck on a peppermint candy." How should the nurse reply? Peppermint has no impact on your intestinal functioning. Peppermint is an antispasmodic and has anti-inflammatory properties. Peppermint is a laxative, which increases motility. Peppermint has antinausea affect and can improve stomach irritation.

Peppermint is an antispasmodic and has anti-inflammatory properties.

Complications of cirrhosis

Portal Hypertension, esophageal/ gastric varices, hepatic encephaopathy

Signs and symptoms of cholecystitis

RUQ pain, rebound tenderness/guarding, fever, tachycardia

Which nursing action, when caring for a patient with abdominal trauma, may result in further complications? Administering IV fluids to the patient Removing objects emerging from the wound Promoting hemodynamic stability Obtaining samples for a complete blood count (CBC), serum electrolytes, and urinalysis

Removing objects emerging from the wound

What lab values indicate malnutrition?

Serum Prealbumin <15 mg/dl. Best marker for Malnutrition. See Prealbumin for interpretation and monitoring. Serum Albumin <3.4 mg/dl. Serum Transferrin <200 mg/dl. Total Lymphocyte Count <1500/mm3. Total Cholesterol <160 mg/dl.

Hypokalemia complications

Severe hypokalemia may manifest as bradycardia with cardiovascular collapse. Cardiac arrhythmias and acute respiratory failure from muscle paralysis are life-threatening complications that require immediate diagnosis

Harold​ Jackson, a​ 65-year-old former elementary school teacher living at​ home, remembers having influenza A in the pandemic of 2009. Which teaching point can the nurse include to decrease Mr.​ Jackson's risk of getting the​ flu?

The injectable vaccine might be less effective at a time when he is at greater risk.-Correct Because he is not in a​ long-term care​ facility, he is unlikely to get the virus. The intranasal spray vaccine would be ideal for his situation. Because he had the most common​ flu, influenza​ A, he has antibodies against it. Because Mr. Johnson is over 65 years​ old, he faces a double​ risk: the injectable vaccine might be less​ effective, and his age increases his risk. Intranasal spray vaccine can be given only to people under age 50. The influenza virus changes every​ year, so people do not have a chance to develop antibodies to fight the current virus. Although people in a​ long-term care facility have a higher risk of contracting the​ virus, living at home is no protection against contact with it.

What are reasons to encourage the client with influenza to stay on bed​ rest? ​(Select all that​ apply.)

To prevent the spread of infection-Correct To boost the immune system-Correct To reduce malaise-Correct To prevent dehydration To reduce cough Bed rest reduces​ malaise, boosts the immune​ system, and prevents the spread of infection. Bed rest does not prevent dehydration or reduce cough.

The clinic nurse is preparing to give an antiviral drug to an older client with chronic obstructive pulmonary disease​ (COPD) and sickle cell disease. Which ordered medication would cause the nurse to notify the​ prescriber?

Zanamivir- Correct Oseltamivir Tamiflu Xanax Rationale The nurse would question the use of zanamivir in a client with COPD because its route of administration is inhalation. Oseltamivir​ (Tamilflu), given​ orally, does not have this caution. Xanax is a medication used to treat anxiety and would not necessitate notifying the health care provider.

infliximab

a monoclonal antibody used more often in Crohn's disease

What is secondary TB

a reactivation of the disease in a previously infected person - usually as a result of lower immunity

Select the correct statements for each question when managing the client with chronic pancreatitis. Select all that apply. Dietary and alcohol intake with chronic pancreatitis a. Avoid coffee and caffeine for prevention of gastric distress b. Low-fat diet required for malabsorption of fats c. Alcohol should be avoided to minimize pancreatic inflammation d. Maintain a diabetic diet for glycemic control e. Increased caloric intake too combat weight loss f. Total parenteral nutrition for nutritional support

a. Avoid coffee and caffeine for prevention of gastric distress b. Low-fat diet required for malabsorption of fats c. Alcohol should be avoided to minimize pancreatic inflammation e. Increased caloric intake too combat weight loss f. Total parenteral nutrition for nutritional support Alcohol, fat, and caffeine in the diet can further exacerbate pancreatic inflammation. A low-fat diet is implemented because pancreatitis leads to malabsorption of fats in the diet. Weight loss is common in chronic pancreatitis, and nutritional support is needed with increased caloric intake and/or TPN therapy.

A community health nurse is planning an educational program about hepatitis A. When preparing the materials, the nurse should identify that which of the following groups is most at risk for developing hepatitis A? a. Children b. Older adults c. Women who are pregnant d. Middle-aged men

a. Children

A nurse is caring for a client who has a history of cirrhosis and is admitted with manifestations of hepatic encephalopathy. The nurse should anticipate a prescription for which of the following laboratory tests to determine the possibility of recent excessive alcohol use? a. Gamma-glutamyl transferase (GGT) b. Alkaline phosphatase (ALP) c. Serum bilirubin d. Alanine aminotransferase (ALT)

a. Gamma-glutamyl transferase (GGT)

The nurse is caring for a client with acute pancreatitis. What complications should the nurse monitor for in the critical client? Select all that apply. a. Hemorrhage b. Gallstone development c. Multiple organ failure d. Pleural effusion e. Osteoporosis

a. Hemorrhage c. Multiple organ failure d. Pleural effusion

Which causes are associated with chronic pancreatitis? Select all that apply. a. Hyperlipidemia b. Cystic fibrosis c. Spicy food indigestion d. Hereditary disorders of the pancreas e. Hypercalcemia

a. Hyperlipidemia b. Cystic fibrosis d. Hereditary disorders of the pancreas e. Hypercalcemia

Select the correct statements for each question when managing the client with chronic pancreatitis. Select all that apply. Pain a. Opioids provide best pain relief b. Non-opioids should be avoided c. Described as persistent and recurring d. Described as right lower quadrant abdominal pain e. Focused over epigastric region f. Pain secondary to impaired pain receptors

a. Opioids provide best pain relief c. Described as persistent and recurring e. Focused over epigastric region Pancreatic pain is described as persistent and recurring in the epigastric or upper left quadrant abdominal area. The pain is in response to inflammation and obstruction of the organ. Opioids provide the best pain relief but non-opioids should be used when pain is less.

A client with chronic pancreatitis may require which type of assistive therapy? a. Pancreatic enzyme replacement therapy b. Plasmapheresis c. Mechanical ventilation d. Hyperbaric oxygen therapy

a. Pancreatic enzyme replacement therapy

A nurse is caring for a client who is scheduled to undergo a liver biopsy for a suspected malignancy. Which of the following laboratory findings should the nurse monitor prior to the procedure? a. Prothrombin time b. Serum lipase c. Bilirubin d. Calcium

a. Prothrombin time

Select the correct statements for each question when managing the client with chronic pancreatitis. Select all that apply. Oral pancreatic enzymes a. Provided in extended-release capsules b. Must be swallowed whole for therapeutic dose c. Helps digest fats and proteins d. Must be taken with to meals e. Do not chew to prevent disruption of enteric coating f. Deliver following ingestion of meals

a. Provided in extended-release capsules b. Must be swallowed whole for therapeutic dose c. Helps digest fats and proteins d. Must be taken with to meals e. Do not chew to prevent disruption of enteric coating Pancreatic enzymes, which help digest fats and proteins, are given with meals and must be swallowed. Administering the medication swallowed whole allows for achievement of the therapeutic dose and prevents disrupting the enteric coating and extended release.

A nurse is caring for a client who is 4 hr postoperative following a laparoscopic cholecystectomy. Which of the following findings should the nurse expect? a. Right shoulder pain b. Urine output 20 mL/hr c. Temperature 38.4° C (101.1° F) d. Oxygen saturation 92%

a. Right shoulder pain

A nurse is caring for a client who has celiac disease. Which of the following foods should the nurse remove from the client's meal tray? a. Wheat toast b. Tapioca pudding c. Hard-boiled egg d. Mashed potatoes

a. Wheat toast

What clinical markers are used in the evaluation of Ranson's Criteria when determining the severity of acute pancreatitis? Select all that apply. a. White blood cell count greater than 16,000/mL b. Rise in hematocrit level greater than 10% c. Fluid sequestration greater than 3 L d. Glucose greater than 200 mg/dL e. Age less than 20 years

a. White blood cell count greater than 16,000/mL d. Glucose greater than 200 mg/dL

What part of the body does Chrons affect?

all of the large intestine

tenesmus

an unpleasant and urgent sensation to deficate

Decreased Hgb

anemia hemorrhage states of hemodilution pregnancy nutritional deficits

Decreased Hct

anemia; overhydration

A client has a​ 6-mm area that is slightly red and soft to the touch at the site of a PPD​ (Mantoux) test. Which finding should the nurse document for this​ client? A. Negative response B. Positive response if the client had an abnormal chest​ x-ray C. Positive response D. Indeterminate response

answer: A

A client with tuberculosis experiences shortness of​ breath, hypoxia,​ cyanosis, and subcutaneous emphysema. Which pathophysiologic change should the nurse suspect as causing this​ client's symptoms? A. Rupture of tuberculosis lesion B. Encapsulation of the bacilli C. Reactivation tuberculosis D. Miliary tuberculosis

answer: A

adalimumab

anti inflammatory given SQ every other week

What is used to treat pneumonia?

antibiotics

What is long treatment for asthma?

atrivant

A nurse is preparing a community education program about hepatitis B. Which of the following statements should the nurse include in the teaching? a. "A hepatitis B immunization is recommended for those who travel, especially military personnel." b. "A hepatitis B immunization is given to infants and children." c. "Hepatitis B is acquired by eating foods that are contaminated during handling." d. "Hepatitis B can be prevented by using good personal hygiene habits and proper sanitation."

b. "A hepatitis B immunization is given to infants and children."

A nurse is teaching a client who has Barrett's esophagus and is scheduled to undergo an esophagogastroduodenoscopy (EGD). Which of the following statements should the nurse include in the teaching? a. "This procedure is performed to measure the presence of acid in your esophagus." b. "This procedure can determine how well the lower part of your esophagus works." c. "This procedure is performed while you are under general anesthesia." d. "This procedure can determine if you have colon cancer."

b. "This procedure can determine how well the lower part of your esophagus works."

Select the correct statements for each question when managing the client with chronic pancreatitis. Select all that apply. Gastrointestinal prophylaxis therapy for chronic pancreatitis a. Used to restore normal bowel function b. Helps decrease gastric acid secretion c. Relies on histamine blockers d. Relies on proton pump inhibitors e. Increases bulk of stools f. Assists in normalization of bowel frequency

b. Helps decrease gastric acid secretion c. Relies on histamine blockers d. Relies on proton pump inhibitors Gastrointestinal prophylaxis therapy consists of using either histamine blockers or proton pump inhibitors in response to increased gastric acid secretion. It has no effect on bowel function or activity in terms of frequency or bulk of stool formation.

A nurse is caring for a client who has a percutaneous endoscopic gastrostomy (PEG) tube and is receiving intermittent feedings. Prior to initiating the feeding, which of the following actions should the nurse take first? a. Flush the tube with water. b. Place the client in semi-Fowler's position. c. Cleanse the skin around the tube site. d. Aspirate the tube for residual contents.

b. Place the client in semi-Fowler's position.

Select the correct statements for each question when managing the client with chronic pancreatitis. Select all that apply. Pancreatic Enzyme Replacement Therapy (PERT) a. Provides glycogen and amylase b. Provides amylase, lipase, and protease c. Used in the treatment of acute pancreatitis d. Helps to aid in absorption of nutrients e. Causes weight loss which helps to normalize insulin levels f. Used to correct problems of malabsorption and malnutrition

b. Provides amylase, lipase, and protease d. Helps to aid in absorption of nutrients f. Used to correct problems of malabsorption and malnutrition PERT therapy is used in the treatment of chronic pancreatitis. It is given to assist in absorption of nutrients in foods and provides amylase, lipase, and protease. It does not cause weight loss.

A nurse is caring for a client who is scheduled to undergo an esophagogastroduodenoscopy (EGD). The nurse should identify that this procedure is used to do which of the following? a. To visualize polyps in the colon b. To detect an ulceration in the stomach c. To identify an obstruction in the biliary tract d. To determine the presence of free air in the abdomen

b. To detect an ulceration in the stomach

What test can differentiate between ulcerative colitis and crohns?

barium enemas

ulcerative colitis location

begins in the rectum and move towards the cecum

How do you know if ulcerative colitis is a reality?

bloody stools

A nurse is caring for a client who has fulminant hepatic failure. Which of the following procedures should the nurse anticipate for this client? a. Endoscopic sclerotherapy b. Liver lobectomy c. Liver transplant d. Transjugular intrahepatic portal-systemic shunt placement

c. Liver transplant

A nurse is assessing a client who was admitted with a bowel obstruction. The client reports severe abdominal pain. Which of the following findings should indicate to the nurse that a possible bowel perforation has occurred? a. Elevated blood pressure b. Bowel sounds increased in frequency and pitch c. Rigid abdomen d. Emesis of undigested food

c. Rigid abdomen

A nurse in the emergency department is caring for a client who has bleeding esophageal varices. The nurse should anticipate a prescription for which of the following medications? a. Famotidine b. Esomeprazole c. Vasopressin d. Omeprazole

c. Vasopressin

What nutritional concern will the nurse expect to find in a client who has chronic pancreatitis? a. Weight gain b. Denies nausea c. Weight loss d. Stabilized weight pattern with different food preferences

c. Weight loss

What are the 4 types of pneumopnia?

community hospital acquired health care acquired ventilator

Hypochloremia Complications

congestive heart failure. prolonged diarrhea or vomiting. chronic lung disease, such as emphysema. metabolic alkalosis, when your blood pH is higher than normal.

What do emphysema sounds sound like?

creaking on a rocking chair

The nurse is providing client teaching on pancreatic enzyme replacement therapy. What should the nurse tell the client this therapy is used to treat? a. Decreased insulin level b. Inflammation and obstruction c. Pseudocyst d. Malnutrition and malabsorption

d. Malnutrition and malabsorption

Which clinical presentation is seen in both acute and chronic pancreatitis? a. Edema b. Hyponatremia c. Hypercalcemia d. Pain

d. Pain

A nurse is caring for a client who is dehydrated and is receiving continuous tube feeding through a pump at 75 mL/hr. When the nurse assesses the client at 0800, which of the following findings requires intervention by the nurse? a. A full pitcher of water is sitting on the client's bedside table within the client's reach. b. The disposable feeding bag is from the previous day at 1000 and contains 200 mL of feeding. c. The client is lying on the right side with a visible dependent loop in the feeding tube. d. The head of the bed is elevated 20°.

d. The head of the bed is elevated 20°

Hypophosphatemia complications

death of muscle tissue (rhabdomyolysis) breathing failure. red blood cell destruction (hemolytic anemia) an irregular heart rhythm (arrhythmia)

Which statement best reflects the difference between acute and chronic pancreatitis? a. The pain from acute pancreatitis is less intense than from chronic pancreatitis. b. Fluid replacement therapy is required only for treatment of acute pancreatitis. c. Once a client has been diagnosed with acute pancreatitis, it will progress rapidly to chronic pancreatitis regardless of the response to treatment intervention. e. Damage to the pancreas can be reversed in acute pancreatitis whereas with chronic pancreatitis both exocrine and endocrine function are compromised.

e. Damage to the pancreas can be reversed in acute pancreatitis whereas with chronic pancreatitis both exocrine and endocrine function are compromised.

How do you determine if a patient is stable?

look at BP

What kind of diet do people with diverticulitis need?

low fiber diet while inflamed

chrons disease location

most often in the terminal illeum with patchy involvement through all layers of the bowel

health care acquired pneumonia

onset occurs less than 48 hours after admission

Complications of pneumonia

pleurisy, atelectasis, pulmonary edema, respiratory failure,

What are the pneumonia vaccines?

pneumovax and prevnar 13

pyrazinamide

treats TB but need to take at least 8 ounces of water

aminosalicylates

treats ulcerative colitis - takes 2-4 weeks to be effective

Sulfasalazine

treats ulcerative colitis - has MANY side effects and needs a folic acid supplement

Which IBD always involves rectum?

ulcerative colitis

glucorticoids

used to decrease inflammation - tapered off and stopped after improvement is seen

ethambutol

used to treat TB - avoid alcohol consumption

isoniazid

used to treat TB - need to take on empty stomach

Signs and symptoms of acute pancreatitis

(1) Abdominal pain, usually in epigastric region, radiating to back in 50% of patients; (2) Often steady, dull, and severe; worse when supine and after meals; (2) Low-grade fever, tachy, hypotensive, leukocytosis; (4) Epigastric tenderness, abdominal distention; (5) Decreased or absent bowel sounds indicate partial ileus; (6) Grey Turner's sign [flank ecchymoses], Cullen's sign [periumbilical ecchymoses] and Fox's Sign [Ecchymoses of inguinal ligament]

surgery for ulcerative colitis

*total colectomy (ileostomy) remove all colon left with the ilium *Kock's ileostomy (nipple valve and opens and closes to empty intestines) or J Pouch (no external bag)

Surgery for Crohn's

-May remove only the affected area -the client may end up with an ileostomy or a colostomy. It just depends on the area affected.

Normal lipase level

0-110 U/L

Normal creatinine levels

0.8-1.4 mg/dL

13. Which sign/symptoms should the nurse expect to find in a client diagnosed with ulcerative colitis? 1. Twenty bloody stools a day 2. Oral temperature of 102 degrees Fahrenheit 3. Hard, rigid abdomen 4. Urinary stress incontinence

1

17. The client diagnosed with IBD is prescribed total parental nutrition (TPN). Which intervention should the nurse implement? 1. Check the client's glucose level 2. Administer an oral hypoglycemic 3. Assess the peripheral intravenous site 4. Monitor the client's oral food intake

1

22. The client diagnosed with Crohn's disease, also known as regional enteritis. Which statement by the client supports thing diagnosis? 1. "My pain goes away when I have a bowel movement." 2. "I have bright red blood in my stool all the time." 3. "I have episodes of diarrhea and constipation." 4. "My abdomen is hard and rigid and I have a fever."

1

58. The nurse is teaching a class on diverticulosis. Which interventions should the nurse discuss when teaching ways to prevent an acute exacerbation of diverticulosis? Select all that apply. 1. Eat a high-fiber diet. 2. Increase fluid intake. 3. Elevate the HOB after eating. 4. Walk 30 minutes a day. 5. Take an antacid every two (2) hours.

1,2,4

Normal magnesium levels

1.5-2.5 mEq/L

How big does the Mantoux test have to be to test positive?

10 mm

Normal BUN levels

10-20 mg/dL

Normal BUN:Creatinine ratio

10:1

Normal serum sodium

135-145 mEq/L

Normal platelet count

150,000-400,000/mm3

15. The client diagnosed with inflammatory bowel disease has a serum potassium level of 3.4 mEq/L. Which action should the nurse implement first? 1. Notify the health-care provider 2. Assess the client fo muscle weakness 3. Request telemetry for the client 4. Prepare to administer potassium IV

2

16. The client is diagnosed with an acute exacerbation of ulcerative colitis. Which intervention should the nurse implement? 1. Provide a low-residue diet 2. Rest the client's bowels 3. Asses vital signs daily 4. Administer antacids orally

2

Normal phosphorus levels

2.5-4.5 mg/dL

Normal amylase level

25-151

How do you know if you are clear of TB?

3 negative consecutive cultures

Decreased serum sodium (hyponatremia)

Acute hyponatremia can lead to much more severe complications such as cerebral edema, brain disease, herniation of the brain, cardiopulmonary arrest, seizure, coma and even death.

normal potassium levels

3.5-5.0 mEq/L

Normal CO2 levels

35-45 mmHg

21 . The client diagnosed with IBD is prescribed sulfasalazine (Asulfidine), a sulfonamide antibiotic. Which statement best describes the rationale for administering this medication? 1. It is administered rectally to help decrease colon inflammation. 2. This medication slows gastrointestinal motility and reduces diarrhea. 3. This medication kills the bacteria causing the exacerbation. 4. It acts topically on the colon mucosa to decrease inflammation.

4

Normal ALT levels

8-20 units/L

Normal calcium levels

8.5-10.5 mg/dL

56. The nurse is preparing to administer a 250-mL intravenous antibiotic to the client. The medication must infuse in one (1) hour. An intravenous pump is not available and the nurse must administer the medication via gravity with IV tubing 10 gtts/min. At what rate should the nurse infuse the medication?_______

42 gtts/min.

The nurse prepares educational material on the development of tuberculosis for a group of new nurses. In which order should the nurse explain the disease process​ develops? - (1) WBCs surround bacteria - (2)Tubercle forms and caseous necrosis occurs -(3)Droplet nuclei enter the lungs and the bacteria multiply -(4)Scar tissue forms around tubercle and the lesion calcifies -(5)Mycobacterium tuberculosis in droplet nuclei enter the air

5, 3, 1, 2, 4,

Normal AST levels

5-40 units/L

Normal WBC range

5000-10000/mm3

A patient, who has recovered from cholecystitis, is being discharged home. What meal options below are best for this patient?* A. Baked chicken with steamed carrots and rice B. Broccoli and cheese casserole with gravy and mashed potatoes C. Cheeseburger with fries D. Fried chicken with a baked potato

A

An elderly client with pneumonia may appear with which of the following symptoms first? A. Altered mental status and dehydration B. Fever and chills C. Hemoptysis and dyspnea D. Pleuritic chest pain and cough

A Fever, chills, hemoptysis, dyspnea, cough, and pleuritic chest pain are common symptoms of pneumonia, but elderly clients may first appear with only an altered mental status and dehydration due to a blunted immune response.

What does elevated RBCs indicate?

A high RBC count may be a result of sleep apnea, pulmonary fibrosis, and other conditions that cause low oxygen levels in the blood. Performance-enhancing drugs like protein injections and anabolic steroids can also increase RBCs. Kidney disease and kidney cancers can lead to high RBC counts as well.

An older adult resident in a long-term-care facility becomes confused and agitated, telling the nurse, "Get out of here! You're going to kill me!" Which action will the nurse take first? A) Check the resident's oxygen saturation. B) Do a complete neurologic assessment. C) Give the prescribed PRN lorazepam (Ativan). D) Notify the resident's primary care provider.

A) Check the resident's oxygen saturation.

A nurse is caring for an 89-year-old client admitted with pneumonia. He has an IV of normal saline running at 100 mL/hr and antibiotics that were initiated in the emergency department 3 hours ago. He has oxygen at 2 liters/nasal cannula. What assessment finding by the nurse indicates that goals for a priority diagnosis have been met for this client? A) The client is alert and oriented to person, place, and time. B) Blood pressure is within normal limits and client's baseline. C) Skin behind the ears demonstrates no redness or irritation. D) Urine output has been >30 mL/hr per Foley catheter

A) The client is alert and oriented to person, place, and time.

The pt with cirrhosis has increased ab girth from ascites. The nurse should know that this fluid gathers in the ad for which reasons? Select all that apply. a. there is decreased colloid oncotic pressure from the liver's inability to synthesize albumin. b. hyperaldosteronsim related to damaged hepatocytes increases sodium and fluid retention c. portal hypertension pushes proteins from the blood vessels, causing leaking into the peritoneal cavity d. osmoreceptors in the hypothalamus stimulate thirst, which causes the stimulation to take in fluid orally e. overactivity of the enlarged spleen results in increased removal of blood cells from the circulation, which decreases the vascular pressure.

A, B, C Ascites related to cirrhosis is caused by decreased colloid oncotic pressure from the lack of albumin form liver inability to synthesize it and the portal hypertension shifts protein into peritoneal cavity, and hyperaldosteronism which increases sodium and fluid retention.

The nurse is caring for a pt who often coughs and chokes while eating and taking his meds, but the pt insists he is fine. The nurse recognizes this as a priority pt problem of risk for aspiration. The nursing interventions she would implement to prevent aspiration pneumonia are Select All That Apply A. HOB always elevated during meals B. Monitor pt ability to swallow small bites C. Give small frequent drinks of thin liquid D. Consult a nutritionist and obtain swallow studies E. Monitor the patient's ability to swallow saliva F. Place pt on NPO status til swallowing returns to normal

A, B, D, E

Nursing management of the patient with acute pancreatitis includes: (SATA) A. Check for signs of hypocalcemia B. Provide a diet low in carbohydrates C. Giving insulin based on sliding scale D. Observing stools for signs of steatorrhea E. Monitoring for infection, particularly respiratory tract infection

A, E Rationale: During the acute phase, it is important to monitor vital signs. Hemodynamic stability may be compromised by hypotension, fever, and tachypnea. Injection fluids are ordered, and the response to therapy is monitored. Fluid and electrolyte balances are closely monitored. Frequent vomiting, along with gastric suction, may result in decreased levels of chloride, sodium, and potassium. Because hypocalcemia can occur in acute pancreatitis, the nurse should observe for symptoms of tetany, such as jerking, irritability, and muscular twitching. Numbness or tingling around the lips and in the fingers is an early indicator of hypocalcemia. The patient should be assessed for Chvostek's sign or Trousseau's sign. A patient with acute pancreatitis should be observed for fever and other manifestations of infection. Respiratory infections are common because the retroperitoneal fluid raises the diaphragm, which causes the patient to take shallow, guarded abdominal breaths.

A pt is suspected on having community-acquired pneumonia. The nurse anticipates which of the following tests to be done to dx pt A. sputum gram stain B. Pulmonary function test C. fluorescein bronchoscopy D. peak flow meter measurement E. chest x-ay

A, E sputum gram stain CXR

A nurse is completing nutritional teaching for a client who has pancreatitis. Which of the following statements by the client indicates an understanding of the teaching? (SATA) A. I plan to eat small, frequent meals. B. I will eat easy-to-digest foods with limited spice C. I will use skim milk when cooking D. I plan to drink regular cola E. I will limit alcohol intake to two drinkers per day

A,B C Rationale: Patients with pancreatitis should eat small, frequent, easy to digest, low-fat meals. Pt should avoid alcohol and caffeinated beverages.

Tests to diagnose cholecystitis

Abdominal X-ray, Abdominal Ultrasound, CT scan, HIDA scan (nuclear medicine uses radioactive tracer to study flow of bile), ERCP (allows visualization of common bile duct), Cholecystography (Radiographic test of gallbladder after patient orally takes radiopaque dye)

Elevated AST and ALT

Abnormal liver function test with raised alanine aminotransferase (ALT) and raised aspartate aminotransferase (AST) are commonly seen in primary care setting. Chronic alcohol consumption, drugs, non-alcoholic steatohepatitis (NASH) and chronic viral hepatitis are common causes associated with raised ALT and AST.

2. The liver receives deoxygenated blood from the: A. Portal vein B. Gastric artery C. Hepatic artery D. Hepatorenal vein

Answer: A

As the nurse reviews the history of a client admitted in​ labor, the nurse notes that the client has inactive tuberculosis. Which should the nurse include in the plan of care for this​ client? A. The client will be allowed to breastfeed the infant. B. Once​ delivered, the infant will be placed on prophylactic treatment. C. Direct contact should be avoided until the client is noninfectious. D. Pharmacologic therapy for the client should be initiated immediately.

Answer: A

Who is more at risk for ulcerative colitis?

Askenazki Jewish individuals

What foods should be avoided in patients with UC?

Avoid fresh fruits, vegatables, prunes, and caffeinated beverages, concentrated sweets, decrease alcohol

The client diagnosed with acute pancreatitis is being discharged home. What statement by the client indicates the teaching has been effective? A. I should decrease my intake of coffee, tea, and cola B. I will eat a low fate diet and avoid spicy food C. I will check my amylase and lipase levels daily D. I will return to work tomorrow but take it easy

B Rationale: High fat and spicy foods stimulate pancreatic enzymes. Caffeinated beverages should be avoided not decreased. There are no daily tests the client can take at home. The client will be fatigued as a result as a lowered metabolic rate and will need to rest.

A nurse evaluates the blood theophylline level of a client receiving aminophylline (theophylline) by intravenous infusion. The nurse would determine that a therapeutic blood level exists if which of the following were noted in the laboratory report? A. 5 mcg/mL B. 15 mcg/mL C. 25 mcg/mL D. 30 mcg/mL

B The therapeutic theophylline blood level range from 10-20 mcg/mL.

Which statement best describes pneumonia? A. an infection of just the windpipe because the lungs are clear of any problems B. a serious inflammation, caused by various things, of the bronchioles C. only an infection of the lungs with mild to severe effects on breathing D. an inflammation resulting from damage to the lungs due to long-term smoking

B a serious inflammation, caused by various things, of the bronchioles

The clinical instructor is reviewing common complications of pneumonia with the students. She knows that further instruction is needed when the students identify which of these? A. sepsis B. ventilation/perfusion issues C. hypoxemia D. pleural effusion E. respiratory failure F. atelectasis

B pneumonia is a ventilation problem not perfusion

The pt with pneumonia has a priority problem of ineffective airway clearance with bronchospasms. Pt has no previous chronic resp disorders. The nurse will obtain an order for which intervention? A. increased liters of humidified oxygen via facemask B. scheduled and prn aerosol nebulizer bronchodilator treatments C. handheld bronchodilator inhaler prn D. corticosteroid via inhaler or IV to reduce inflammation

B scheduled and prn aerosol nebulizer bronchodilator treatments

Which pt is at higher risk for developing pneumonia? A. any hospitalized pt between 19 - 64 y.o. B. 36 y.o. trauma pt on mechanical ventilator C. disabled 51 y.o. with abdominal pain, d/c home D. Any pt who has not received the pneumonia vaccine

B 36 y.o. trauma pt on mechanical ventilator

The home health nurse is visiting the home of an older adult pt recovering from a knee replacement. She ids a priority pt problem of risk for respiratory infection. Which is a normal aging factor contributing to the risk A. inability of a forced cough B. decreased strength of resp. muscles C. increased macrophages in alveoli D. increased elastic recoil of alveoli

B decr strength resp muscles

A patient in the emergency room has signs and symptoms associated with cholecystitis. What testing do you anticipate the physician will order to help diagnose cholecystitis? Select all that apply:* A. Lower GI series B. Abdominal ultrasound C. HIDA Scan (Hepatobiliary Iminodiacetic AciD scan) D. Colonoscopy

B and C.

A client with pneumonia caused by aspiration after alcohol intoxication has just been admitted. The client is febrile and agitated. Which health care provider order should the nurse implement first? A) Administer levofloxacin (Levaquin) 500 mg IV. B) Draw aerobic and anaerobic blood cultures. C) Give lorazepam (Ativan) as needed for agitation. D) Refer to social worker for alcohol counseling.

B) Draw aerobic and anaerobic blood cultures.

pneumonia may present differently in the older adult than in the younger adult? A) Crackles on auscultation B) Fever C) Headache D) Wheezing

B) Fever

An older client presents to the emergency department with a 2-day history of cough, pain on inspiration, shortness of breath, and dyspnea. The client never had a pneumococcal vaccine. The client's chest x-ray shows density in both bases. The client has wheezing upon auscultation of both lungs. Would a bronchodilator be beneficial for this client? A) It would not be beneficial for this client. B) It would help decrease the bronchospasm. C) It would clear up the density in the bases of the client's lungs. D) It would decrease the client's pain on inspiration.

B) It would help decrease the bronchospasm.

Which teaching is essential for a patient who has had a permanent pacemaker inserted? A. Avoid talking on a cell phone. B. Avoid operating electrical appliances over the pacemaker. C. Avoid sexual activity. D. Do not take tub baths.

B. Avoid operating electrical appliances over the pacemaker.

Your patient is diagnosed with acute cholecystitis. The patient is extremely nauseous. A nasogastric tube is inserted with GI decompression. The patient reports a pain rating of 9 on 1-10 scale and states the pain radiates to the shoulder blade. Select all the appropriate nursing interventions for the patient:* A. Encourage the patient to consume clear liquids. B. Administered IV fluids per MD order. C. Provide mouth care routinely. D. Keep the patient NPO. E. Administer analgesic as ordered. F. Maintain low intermittent suction to NG tube.

B, C, D, E, and F.

Which statements below are CORRECT regarding the role of bile? Select all that apply:* A. Bile is created and stored in the gallbladder. B. Bile aids in digestion of fat soluble vitamins, such as A, D, E, and K. C. Bile is released from the gallbladder into the duodenum. D. Bile contains bilirubin.

B, C, and D

What client problem has a priority for the client diagnosed with acute pancreatitis? A. Risk for fluid volume deficient B. Alteration in comfort C. Imbalanced nutrition: less than the boy requires D. Knowledge deficient

B. Rationale: Autodigestion of the pancreas results in severe pain, accompanied by nausea, vomiting,m abdominal tenderness, and muscle guarding. The Maslow's hierarchy should be applied. After ABC's pain is the next priority

A patient with sudden pain in the left upper quadrant radiating to the back and vomiting was diagnosed with acute pancreatitis. Which intervention should the nurse include in the patient's plan of care? A) Immediately start enteral feeding to prevent malnutrition. B) Insert an NG and maintain NPO status to allow pancreas to rest. C) Initiate early prophylactic antibiotic therapy to prevent infection. D) Administer acetaminophen (Tylenol) every 4 hours for pain relief.

B. Rationale: Initial treatment with acute pancreatitis will include an NG tube if there is vomiting and being NPO to decrease pancreatic enzyme stimulation and allow the pancreas to rest and heal. Fluid will be administered to treat or prevent shock. The pain will be treated with IV morphine because of the NPO status. Enteral feedings will only be used for the patient with severe acute pancreatitis in whom oral intake is not resumed. Antibiotic therapy is only needed with acute necrotizing pancreatitis and signs of infection.

What is portal hypertension?

Because fibrotic liver tissue develops in cirrhosis, the liver essentially becomes resistant to normal portal venous flow. Venous pressure in the liver area begins to climb as venous flow tries to overcome the resistance. Venous back pressure increases, veins going into the liver begin to dilate, the liver and the surrounding area begin to be engorged with larger than normal amounts of venous blood.

When is the best time to get a sputum culture?

Before antibiotics are given, in the morning, as soon as possible

How are Crohn's disease and ulcerative colitis similar?

Both involve inflammation of the bowel, and can lead to very similar complications/conditions

The nurse is assessing for appendicitis at McBurney's point. Which location is this on the image?

Bottom of upper-right quadrant to halfway down lower-right quadrant

What is hepatic encephalopathy?

Brain dysfunction and damage caused by increased ammonia in the blood, resulting from severe liver disease

Treatments for Diverticulitis

Broad spectrum antibiotics (Cipro, Metronidazole, Augementin, Amoxicillin), clear fluid diet, IV fluids, NPO and bowel rest

The physician orders a patient's T-Tube to be clamped 1 hour before and 1 hour after meals. You clamp the T-Tube as prescribed. While the tube is clamped which finding requires you to notify the physician?* A. The T-Tube is not draining. B. The T-Tube tubing is below the patient's waist. C. The patient reports nausea and abdominal pain. D. The patient's stool is brown and formed.

C

You're precepting a nursing student who is helping you provide T-Tube drain care. You explain to the nursing student that the t-shaped part of the drain is located in what part of the biliary tract?* A. Cystic duct B. Common hepatic duct C. Common bile duct D. Pancreatic duct

C

Your recent admission has acute cholecystitis. The patient is awaiting a cholecystostomy. What signs and symptoms are associated with this condition? Select all that apply:* A. Right lower quadrant pain with rebound tenderness B. Negative Murphy's Sign C. Epigastric pain that radiates to the right scapula D. Pain and fullness that increases after a greasy or spicy meal E. Fever F. Tachycardia G. Nausea

C, D, E, F, and G.

You're providing a community in-service about gastrointestinal disorders. During your teaching about cholecystitis, you discuss how cholelithiasis can lead to this condition. What are the risk factors for cholelithiasis that you will include in your teaching to the participants? Select all that apply:* A. Being male B. Underweight C. Being female D. Older age E. Native American F. Caucasian G. Pregnant H. Family History I. Obesity

C, D, E, G, H and I

A nurse is assessing a client who has pancreatitis. Which of the following actions should the nurse take to assess the presence of Cullen's sign. A. Tap lightly at the costovertebral margin on the client's back B. Palpate the RLQ C. Inspect the skin around the umbilicus D. Auscultate the area below the scapula

C. Rationale: Cullen's sign is indicated by a bluish-gray discoloration in the periumbillical area

The client is admitted to the medical department with a diagnosis of R/O acute pancreatitis. What laboratory values should the nurse monitor to confirm this diagnosis? A. Creatinine and BUN B. Troponin and CK-MB C. Serum amylase and lipase D. Serum bilirubin and calcium

C. Rationale: Serum amylase levels increase within two to 12 hours of the onset of acute pancreatitis; lipase elevates and remains elevated for seven to 14 days

The school nurse is talking to fifth graders about the use of tissues when blowing​ one's nose. Which cause of a runny nose should the nurse include in the teaching​ session?

Cells that line the breathing passages dying and releasing fluid-Correct Not enough coughing and sneezing to clear the nose Drinking lots of water when the weather turns cold Solid viruses becoming fluid when fever melts them Rationale A runny nose is caused by cells that line the breathing passages dying and releasing fluid. It is not a matter of​ fever, cold​ weather, or coughing and sneezing.

A client with irritable bowel syndrome is instructed to begin using a food diary. The client is unclear as to why a food diary is needed. What is the basis of the nurse's explanation? Flare-ups of the disease process are associated with increased caloric intake. Food selection should be based on following a low-sodium, low-fat diet. Excess fluid intake can lead to exacerbations of the disease. Certain foods can act as triggers and cause flare-ups of the disease.

Certain foods can act as triggers and cause flare-ups of the disease.

Hyperkalemia complications

Complications of hyperkalemia range from mild ECG changes to cardiac arrest. Weakness is common as well. The primary cause of morbidity and mortality is potassium's effect on cardiac function. The mortality can be as high as 67% if severe hyperkalemia is not treated rapidly.

Which is true of ulcerative colitis? Select all that apply. Complications such as hemorrhage and nutritional deficiencies may occur. It occurs anywhere in the gastrointestinal tract from mouth to anus. Weight loss rarely occurs. Granuloma is common. Stools are frequent and watery with blood and mucus.

Complications such as hemorrhage and nutritional deficiencies may occur. Weight loss rarely occurs. Stools are frequent and watery with blood and mucus.

The gallbladder is found on the __________ side of the body and is located under the ____________. It stores __________.* A. right; pancreas; bilirubin B. left; liver; bile C. right; thymus' bilirubin D. right; liver; bile

D

The nurse notices a visitor walking into the room of a pt on airborne isolation with no protective gear. What does the nurse do? A. Ensures that the pt is wearing a mask B. Tells the visitor that the pt cannot receive visitors at this time C. Gives a particulate air respirator to the visitor D. Gives a mask to the visitor

D Because the visitor is entering the pt's isolation environment, the visitor must wear a mask. The pt typically must wear a mask only when he or she is outside of an isolation environment. Turning the visitor away is inappropriate and unnecessary. It would not be necessary for the visitor to wear an air respirator.

A nurse is providing dietary teaching for a client who has chronic pancreatitis. Which of the following food selections by the client indicates an understanding of the teaching? A. 8 oz whole milk B. One slice of beef bologna C. 1 oz cheddar cheese D. 1 cup sliced banana

D. 1 cup sliced banana Foods that are high in fat can cause diarrhea for clients who have pancreatitis. One cup of sliced banana, which contains 0.49 g of fat, is a low-fat food option. Clients who have pancreatitis should consume a high-protein and low-fat diet with an adequate amount of carbohydrates and calories.

What are esophageal varices?

Dilated submucosal veins in the lower esophagus

A nurse is completing an admission assessment of a client who has pancreatitis. Which of the following findings should the nurse expect? A. Pain in the UQ rating to the shoulder B. Report of pain being worse when sitting upright C. Pain relieved with defecation D. Epigastric pain radiating to the left shoulder

D. Rationale: A client with pancreatitis will report pain being worse when lying down in the fetal position, and pain that radiates to the back. left flank, or left, shoulder.

The nurse is completing discharge teaching to the client diagnosed with acute pancreatitis. What instruction should the nurse discuss with the client? A. Instruct the importance to avoid all stress B. Explain the correct way to take pancreatic enzymes C. Instruct the client to decrease alcohol intake D. Discuss the importance of stopping smoking

D. Rationale: Alcohol must be avoided completely due to its destruction of the pancreas. Stress stimulates the liver, but it is unrational to avoid all stress. Pancreatic enzymes are only needed for chronic pancreatitis. Smoking stimulates the pancreas to release pancreatic enzymes.

A nurse is completing the admission assessment of a client who has acute pancreatitis. Which finding is the first priority? A) History of cholelithiasis B) Elevated serum amylase levels C) Decrease in bowel sounds upon auscultation D) Hand spasms present when blood pressure is checked

D. Rationale: the greatest risk to the client is ECG changes and hypotension from hypocalcemia. Hand spasms when taking blood pressure is a manifestation of hypocalcemia

The nurse is teaching a patient with a new permanent pacemaker. Which statement by the patient indicates a need for further discharge education? A. "I will be able to shower again soon." B. "I need to take my pulse every day." C. "I might trigger airport security metal detectors." D. "I no longer need my heart pills."

D. "I no longer need my heart pills."

The nurse is providing teaching to a client who is postoperative following a rhinoplasty. Which of the following instructions should the nurse include? A. "Apply warm compresses to the face." B. "Take aspirin 650 milligrams by mouth for mild pain." C. "Close your mouth when sneezing." D. "Lie on your back with your head elevated 30° when resting."

D. "Lie on your back with your head elevated 30° when resting." - The nurse should instruct the client to rest in the semi-fowler's position to prevent aspiration of nasal secretions.

Kaitlin has provided the nurse with a food diary to help identify possible food triggers for IBS. Which diet entry poses a concern? Select all that apply. Egg substitute omelet for breakfast Diet sodas at lunch time Bran cereal for breakfast English breakfast decaffeinated tea for breakfast Apple as an afternoon snack

Diet sodas at lunch time Apple as an afternoon snack

What lab values indicate cholecystitis?

Elevated liver enzymes (AST/ALT), elevated bilirubin (>1.2)

Kaitlin reports clinical symptoms that affect her enjoyment of meals. Which symptoms are associated with IBS? Eructation and dyspepsia Hiccups and indigestion Dizziness and feelings of satiety Urge to void

Eructation and dyspepsia (belching and heartburn)

After a week of caring for a client with the​ flu, the home care nurse modifies a nursing intervention concerning​ fluids; adds a new nursing diagnosis and intervention related to ineffective airway​ clearance; and discontinues a nursing diagnosis of fatigue. Which phase of the nursing process does this activity​ represent?

Evaluation- Correct Assessment Implementation Planning Rationale The evaluation phase of the nursing process makes those changes in the plan of care.​ Assessment, planning, and implementation precede that phase.

What are the risk factors for liver and pancreatic disorders?

Excessive and/or long term use of alcohol

Normal RBC range

Female: 4.2-5.4 million Male: 4.7-6.2 million

Kaitlin tells the nurse she feels she is losing her friends because her dietary restrictions and symptoms are preventing her from socializing. What might the nurse suggest? Allow for cheat meals Find an online support group Find new friends Ignore restrictions when out with friends

Find an online support group

Treatments for Ulcerative Colitis and Crohn's

Fluid and electrolyte management, Rest, medications, Colectomy (for Crohn's)

What are ascites?

Fluid in peritoneal cavity due to portal hypertension

decreased BUN/creatinine ratio

Fluid volume excess or acute renal tubular acidosis

Which herbal therapy would aid in the expulsion of flatulence in a client diagnosed with irritable bowel syndrome? Ginger St. John's Wort Garlic Echinacea

Ginger

Which describes a patient with internal hemorrhoids that prolapse upon defecation and must be reduced manually? Grade I Grade II Grade III Grade IV

Grade III

Kaitlin asks about using alternative therapies to relieve clinical symptoms of IBS. Which suggestions should the nurse make? Select all that apply. Herbal tea Ginger Fennel Peppermint Garlic

Herbal tea Ginger Fennel Peppermint

What are the risk factors for TB?

Homelessness and incarcerated populations, persons with HIV/AIDS

The infection control nurse is talking about the history of the H1N1 influenza. The nurse points out that the virus had genes from more than one source. What sources contributed to the H1N1​ virus? (Select all that​ apply.)

Human genes-Correct Bird genes-Correct Genes from flu viruses found in pigs-Correct Genes from flu viruses found in cats Pig genes Rationale Bird​ genes, human​ genes, and genes from flu viruses found in pigs combined to create the H1N1 virus. Neither pig genes nor genes from flu viruses found in cats were part of the picture.

Treatment of liver cirrhosis

Identify the cause. Management of ascites, respiratory compromise or abdominal pain. Support- prevent further liver damage. Eliminate ETOH, hepatotoxins, exposure to environmental chemicals. Bed rest. Strict I&O Diuretics (assess dehydration, electrolyte imbal., decreased renal function) Paracentesis

remicane

Immunoglobulin G

During which phase of the nursing process would the nurse administer antiviral​ drugs?

Implementation-Correct Diagnosis Assessment Planning The nurse carries out the collaborative nursing intervention of administering antiviral drugs during the implementation phase of the nursing process. The other answer choices precede that phase.

Azathioprine

Imuran

What types of precautions are needed to prevent the spread of influenza and pneumonia?

In the hospital: Both are placed on airborne and contact precautions (mask, gown, gloves, normal air flow) Non-hospital: Get a yearly flu vaccine, get a pneumonia vaccine, stay away from others if you know you are sick

A nurse is discussing postsurgery care techniques for a patient with a hernia. Which techniques are appropriate? Select all that apply. Inspect the incision for signs and symptoms of infection and report any concerns to the primary healthcare provider. Avoid heavy lifting for several weeks. Promote atelectasis and venous thromboembolism (VTE). Avoid deep breathing and early ambulation. Splint the surgical site with pillows while coughing.

Inspect the incision for signs and symptoms of infection and report any concerns to the primary healthcare provider. Avoid heavy lifting for several weeks. Splint the surgical site with pillows while coughing.

What is asterixis?

Involuntary jerking movements of the hands.

Treatment of TB

Isoniazid Rifampin Pyrazinamide Streptomycin

Signs and symptoms of Diverticulitis

LLQ pain, high WBC, blood in stool, distention and nausea/vomiting. CT scan with stool, liver, blood and urine tests to check for signs of infection. Antibiotics as possible treatment depending on severity, insertion of tube for drainage or surgery to correct if needed.

What is the treatment for hepatic encephalopathy?

Lactulose - prevents absorption of ammonia Neomycin - kills bowel flora, so decreases ammonia production by intestinal bacteria Diet: low protein

nutrition recommendations for patient with gallbladder/liver issues

Liver needs low protein diet, gallbladder needs low fat diet

The nurse is reviewing the lab results for a pt with cirrhosis and notes that the ammonia level is elevated. Which diet does the nurse anticipate to be presribed for this pt? Low-protein High-protein Moderate-fat High-carb

Low-protein diet Protein provided by the diet is transported to the liver via the portal vein. The liver breaks down protein, which results in the formation of ammonia.

Normal Hct values

Male: 42-52% Female: 37-47%

What is the name of the test given to test for TB?

Mantoux test

Normal Hgb levels

Men: 14-18 Women: 12-16

What lab values needs to be monitored in a patient with pancreatic issues?

Monitor serum lipase/amylase (elevated), elevated BUN, Elevated ALT, Elevated WBC

Which interventions should a nurse implement for a patient after colorectal surgery? Select all that apply. Take vital signs of the patient every 48 hours. Monitor white blood cell count every day for the first few days. Check if the urine output is at least 10 mL/hour during the first 24 hours. Check if the stoma is reddish pink and moist. Ensure that the patient does not receive food and water per order.

Monitor white blood cell count every day for the first few days. Check if the stoma is reddish pink and moist. Ensure that the patient does not receive food and water per order.

What pain medication is contraindicated in patients with cholecystitis?

Morphine

What is the fastest test for TB?

NAAT - nucleic acid amplification test used in respiratory secretions

Treatment for cholecystitis

NPO, IVF, meperidine, metro and gen 3 cephalosporin, laproscopic surgery

Hypocalcemia complications

Neurologic complications of hypocalcemia include acute seizures or tetany, basal ganglia calcification, parkinsonism, hemiballismus, and choreoathetosis. Although some patients with hypocalcemia may improve with treatment, the calcification typically is not reversible.

Do patients sound better right after taking a bronchodialtor?

No - they sound worse before getting better

Hypercalcemia complications

Osteoporosis. ... Kidney stones. ... Kidney failure. ... Nervous system problems. ... Abnormal heart rhythm (arrhythmia).

Is a colostomy permanent or temporary in diverticulitis?

Normally temporary, as long as the surgery goes well. It is only in place until the bowel is properly healed; may be permanent if bowel does not heal adequately after surgery

Which members of the interdisciplinary team would a nurse include in discharge planning for a client who has been diagnosed with irritable bowel syndrome? Select all that apply. Nutrition counseling Psychological therapy Support group Hospice Ostomy support group

Nutrition counseling Psychological Therapy Support group

Risk factors for cholecystitis

Obesity, rapid weight loss, weight loss surgery, high fat diet, genetics, medications

What word is used to describe the most severe spread of​ influenza?

Pandemic-Correct Antigenic Outbreak Epidemic A pandemic is a global epidemic. That makes it more severe than either an epidemic or an​ outbreak, which are synonyms. Antigenicis the adjectival form of​ "antigen."

Complications of IBD

Perineal abscess, Fistula, Strictures

Signs and symptoms of Ulcerative colitis/Crohn's

Persistent diarrhea, abdominal pain/cramps, fever, weight loss, fluid imbalance, malnutrition, mouth ulcers, anemia, blood from rectum

Signs and symptoms of chronic pancreatitis

Persistent or reoccuring episodes of upper abdominal pain radiating to the back. anorexia. nausea and vomiting. weight loss. steatorrhea

What types of precautions are needed to prevent the spread of TB?

Place patient on droplet precautions, yearly skin tests/skin test if known to be in contact with TB, teach to be consistent with medication regimen

What are the different types of TB?

Primary (not infectious) Secondary (Infectious, present symptoms)

Signs and symptoms of TB

Primary: asymptomatic and not infectious Symptomatic TB infection: Fatigue, weight loss, night sweats, cough, rust colored/blood streaked sputum, dyspnea, orthopnea, rales

The caregiver of an older client with flu reports to a geriatric nurse that the client has shallow respiration. What advice can the geriatric nurse give the caretaker to improve the client's ease of breathing?

Put a humidifier at the​ client's bedside.-Correct Elevate the head of the​ client's bed.-Correct Use a face mask. Give antipyretic medications. Give analgesic medications. Rationale Elevating the head of the bed will put the client in an upright position and improve the ease of breathing. A​ humidifier, which increases the water content of inhaled​ air, will also improve the ease of breathing. Using a face mask prevents the spread of infection but does not ease breathing. Giving antipyretic or analgesic medications helps with​ comfort, not breathing.

What can be done to prevent pneumonia and influenza?

Receive vaccinations, stay away from those who are sick; for pneumonia, take full round of antibiotics, if recovering from surgery, make sure to deep breath and move around to prevent fluid buildup

What labs are monitored in patients with liver disorders?

Serum ammonia (elevated, >45), liver enzymes (AST/ALT, AST >40, ALT >56), Serum sodium (can become hyponatremic, <135)

Signs and symptoms of liver cirrhosis

SOB Jaundice Increased abdominal girth Abdominal pain/bloating Enlarged spleen Elevated liver enzymes Increased risk of bleeding Thrombycytopenia Prolonged PT Hemorrhoids Elevated serum ammonia levels Changes in consciousness Changes in motor function Hyponatremia

What foods should be avoided in Crohn's?

Same as UC (low residue), plus, if patient has strictures, need to avoid seeds, nuts, kernels

The nurse is providing teaching to a client with irritable bowel syndrome. Along with teaching the client about proper nutrition, what other topics should the nurse address? Select all that apply. Smoking cessation Keeping daily activity low, exercising only as necessary Sleeping seven to eight hours each night Limiting alcohol intake Investigating alternative therapies

Smoking cessation Sleeping seven to eight hours each night Limiting alcohol intake Investigating alternative therapies

Kaitlin asks the nurse about stress management to help control her symptoms. Which suggestions should the nurse make? Select all that apply. Take a daily walk Go to church every week Consider a group yoga class Find a massage therapist Take 15 minutes each day to meditate

Take a daily walk Consider a group yoga class Find a massage therapist Take 15 minutes each day to meditate

Emily​ Yi, a​ 27-year-old real estate​ agent, visited a clinic to rule out any bacterial complications after her recovery from influenza. The clinic nurse educated her about​ over-the-counter cough​ suppressants, including the importance of limiting their use. What is the​ nurse's concern?

The drug could reduce the effectiveness of coughing during the day. -Correct If taken​ daily, cough suppressants are potentially addictive. The drug could make her too sleepy during the day and keep her awake at night. The empty calories found in​ pleasant-tasting syrups might cause weight gain. The nurse is concerned about promoting airway​ clearance, which is helped by coughing during the day. The nurse is not worried about addiction​ potential, weight​ gain, or sleep pattern alterations.

The public health nurse is educating social workers working with​ low-income families about the reasons to encourage their clients to get an annual flu shot. What reasons should the nurse include in the educational​ session? (Select all that​ apply.)

The predominant flu virus strain changes from year to year. Correct The new vaccine has specific antigens predicted for that year. Correct ​Infants, young​ children, and people aged 50 or older are more likely to get the flu. Correct People without health insurance are at higher risk of getting the flu. People living in apartment buildings are at higher risk of getting the flu. Rationale The predominant flu virus strain changes from year to year. The new vaccine has specific antigens predicted for that year.​ Infants, young​ children, and people aged 50 or older are more likely to get the flu. People living in apartment buildings or who lack health insurance are not at increased risk of getting the flu.

Why is it important to obtain a sputum culture before antibiotics are given?

To ensure that the patient is receiving the correct type of antibiotic; usually will be started on broad spectrum, may be changed to narrow spectrum

which IBD can be cured?

Ulcerative colitis; cured through removal of effected part of the colon

What kind of isolation do pneumonia patients need to be put on?

none - take droplet precautions

An adolescent client with several flu symptoms calls the advice nurse to ask about using cough suppressants. What suggestion from the nurse would promote better airway​ clearance?

Use cough suppressants during the night-Correct Use cough suppressants in the afternoon Use cough suppressants in the evening Use cough suppressants in the morning Rationale To promote better airway​ clearance, the client should use cough suppressants only during the​ night, to allow rest. Their use during the day inhibits normal coughing meant to clear the airway.

Viral vs bacterial pneumonia

Viral pneumonia tends to produce a non-productive cough or clear sputum. Bacterial pneumonia, however, causes productive cough that may produce white, yellow, or green sputum.

In which way are external hemorrhoids diagnosed? Anoscopy Visual inspection Digital examination Sigmoidoscopy

Visual inspection

Elevated amylase and lipase levels

When levels of lipase and amylase are higher than normal it may indicate pancreatic injury or another disease. Most studies show that levels of greater than three times the upper limit of normal usually lead to a diagnosis of pancreatitis, according to guidelines from the American College of Gastroenterology (ACG).

When do you give antibioitics?

after collecting a sputum culture

In which diagnostic study does the patient swallow a pill to capture the image of the gastrointestinal (GI) tract to assess for colorectal cancer? Lower GI series Wireless capsule endoscopy Virtual colonoscopy Sigmoidoscopy

Wireless capsule endoscopy

A strain of influenza is named​ A/Taiwan/89. What does the 89 stand​ for?

Year the strain was identified-Correct Percentage of effectiveness of the flu vaccine Number of people who died from it Number of places it was found in Taiwan This third part of the label contains the year the strain was identified. It is not about the vaccine​'s ​effectiveness, number of people​ dying, or number of places it was found.

Which antiviral drugs are recommended by the Centers for Disease Control and Prevention​ (CDC) for the treatment or prophylaxis of​ influenza? ​(Select all that​ apply.)

Zanamivir-Correct Amantadine Rimantadine Ribavirin Oseltamivir-Correct The CDC currently recommends zanamivir and oseltamivir for the flu. The other antiviral drugs are not recommended by the CDC.

Select the correct statements for each question when managing the client with chronic pancreatitis. Select all that apply. Lab findings with chronic pancreatitis a. Elevated amylase and lipase levels b. Decreased alkaline phosphatase c. Decreased bilirubin d. Elevated glucose level e. Decreased potassium level f. Elevated sodium level

a. Elevated amylase and lipase levels d. Elevated glucose level Amylase and lipase levels are usually elevated, as are levels of serum alkaline phosphatase and bilirubin. As the disease progresses and impairs endocrine functions of the pancreas, serum glucose levels will increase due to the lack of insulin. Potassium and sodium are not typically impacted from the pancreatitis itself.

What do crackles in the lung mean?

air is flowing through liquid or secretions

What is short treatment for asthma?

albuterol

How many spots on the back do you have to listen to?

at least 3

A nurse is assessing a client who is in the early stages of hepatitis A. Which of the following manifestations should the nurse expect? a. Jaundice b. Anorexia c. Dark urine d. Pale feces

b. Anorexia

A nurse is caring for a client who has acute pancreatitis. Which of the following serum laboratory values should the nurse anticipate returning to the expected reference range within 72 hr after treatment begins? a. Aldolase b. Lipase c. Amylase d. Lactic dehydrogenase

c. Amylase

A client with a history of chronic pancreatitis presents to the hospital for admission. The nurse should expect to find which clinical manifestations on assessment? Select all that apply. a. Decreased respiratory rate b. Stable blood pressure c. Clay-colored stools d. Epigastric and left upper quadrant pain e. Steatorrhea

c. Clay-colored stools d. Epigastric and left upper quadrant pain e. Steatorrhea

A nurse is providing dietary teaching to a client who has diverticulitis about preventing acute attacks. Which of the following foods should the nurse recommend? a. Foods high in vitamin C b. Foods low in fat c. Foods high in fiber d. Foods low in calories

c. Foods high in fiber

Elevated Hgb

caused by polycythemia or dehydration

During assessment of a pt with obstructive jaundice, the nurse would expect to find: clay colored stools dark urine and stool pyrexia and pruritis elevated urinary urobilinogen

clay colored stool

A nurse is completing a history and physical assessment for a client who has chronic pancreatitis. Which of the following findings should the nurse identify as a likely cause of the client's condition? a. High-calorie diet b. Prior gastrointestinal illnesses c. Tobacco use d. Alcohol use

d. Alcohol use

A nurse is assessing a client who is experiencing perforation of a peptic ulcer. Which of the following manifestations should the nurse expect? a. Increased blood pressure b. Decreased heart rate c. Yellowing of the skin d. Boardlike abdomen

d. Boardlike abdomen

A nurse is caring for a client who is 2 days postoperative following a gastric bypass. The nurse notes that bowel sounds are present. Which of the following foods should the nurse provide at the initial feeding? a. Vanilla pudding b. Apple Juice c. Diet ginger ale d. Clear liquids

d. Clear liquids

A nurse is caring for a client who is receiving total parenteral nutrition (TPN) therapy and has just returned to the room following physical therapy. The nurse notes that the infusion pump for the client's TPN is turned off. After restarting the infusion pump, the nurse should monitor the client for which of the following findings? a. Hypertension b. Excessive thirst c. Fever d. Diaphoresis

d. Diaphoresis

Which bed assignment would the admission nurse implement for a client being admitted from the emergency department with acute pancreatitis? a. Surgical floor in a private room b. Medical floor in a semi-private room by the nurse's station c. Coronary cardiac care unit d. Intensive care unit

d. Intensive care unit

Elevated Hct

dehydration

Elevated WBC count

infection

Where is the Mantoux test given

intradermally in the forearm

What is the drug regimine for TB?

isoniazi rifampin pyrazinamide ethambutol

Hyperchloremia complications

kidney stones. hampered ability to recover if you have kidney injuries. kidney failure. heart problems. muscle problems. bone problems. coma. death.

Immunomodulators

not effective in treating ulcerative colitis alone - it alters the immune response

hospital acquired pneumonia

onset occurs greater than 48 hours after admission

Which pnemonia vaccine do you get first?

prevnar 13 than 6-12 months later pneumovax

What is pulmonary empyema?

pus in the pleural space

What is the first assessment for respiratory patients?

respiratory rate

What are the cardinal signs of peritonitis?

rigid boardlike abdomen abdominal pain tenderness distension wbc >20000

Hypomagnesemia complications

seizures. cardiac arrhythmias (abnormal heart patterns) coronary artery vasospasm. sudden death.

Hyperphosphatemia complications

short-term complications of hyperphosphatemia include acute hypocalcemia with possible tetany and, more rarely, acute deposition of calcium/phosphate complexes into joints, as well as subcutaneous tissue and other areas of soft tissue

How do you treat ulcerative colitis?

take out the part that is affected

What is the ideal time for a sputum culture?

the morning

miliary

the spread of TB throughout the body when a large number of organisms enter the blood

What does wheezing mean?

there is an obstruction in the airway

Decreased platelets

thrombocytopenia; excessive bleeding; hemophilia

Elevated platelets

thrombocytosis

rifampin

treats TB turns urine orange

What are key assessments for TB?

weight loss lethargy low grade fever night sweats

How do you cure chrons disease?

you don't

What happens with the BCG vaccine?

you will test positive for the TB test if it was in the last 10 years

Signs and symptoms of influenza

~2 day incubation period Headache, fever, sore throat, muscle pain; peaks in 6-12 hours; dry cough develops and worsens over a few days Acute symptoms last ~1 week; lingering cough, fatigue, weakness last additional days or weeks


संबंधित स्टडी सेट्स

Chapter 4: Cultural & Developmental Processes

View Set

Anatomy: Self Study: bones and joints

View Set

Western Center January 2019 questions

View Set

Sociology: Race, Ethnicity, Gender, & Class (Eighth edition)

View Set

Stupid Invisible Man Applied Practice

View Set

Week 2: Introduction to Managerial Accounting and Cost Concepts

View Set

MKT 305-marketing discrepancies and separations

View Set

Chapter 19: Nursing Management of Pregnancy at Risk: Pregnancy-Related Complications

View Set